You are on page 1of 72

1.

OPTIMA REALTY CORPORATION,


EXCLUSIVE CARS, INC., Respondent.

Petitioner,

v.

HERTZ

PHIL.

G.R. No. 183035


January 9, 2013
SERENO, CJ.
FACTS:
Optima is engaged in the business of leasing and renting out commercial
spaces and buildings to its tenants. It entered into a Contract of Lease with
respondent over a 131-square-meter office unit and a parking slot in the
Optima Building for a period of three years commencing on 1 March 2003 and
ending on 28 February 2006. The parties amended their lease agreement by
shortening the lease period to two years and five months. As a result, Hertz
alleged that it experienced a 50% drop in monthly sales and a significant
decrease in its personnels productivity. It then requested a 50% discount on
its rent for the months of May, June, July and August 2005. On 8 December
2005, Optima granted the request of Hertz. However, the latter still failed to
pay its rentals for the months of August to December of 2005 and January to
February 2006, or a total of seven months .Optima wrote another letter to
Hertz, reminding the latter that the Contract of Lease could be renewed only
by a new negotiation between the parties and upon written notice by the
lessee to the lessor at least 90 days prior to the termination of the lease
period. As no letter was received from Hertz regarding its intention to seek
negotiation and extension of the lease contract within the 90-day period,
Optima informed it that the lease would expire on 28 February 2006 and
would not be renewed. On 21 December 2005, Hertz wrote a letter belatedly
advising Optima of the formers desire to negotiate and extend the lease.
However, as the Contract of Lease provided that the notice to negotiate its
renewal must be given by the lessee at least 90 days prior to the expiration
of the contract, petitioner no longer entertained respondents notice. Hertz
filed a Complaint for Specific Performance, Injunction and Damages and/or
Sum of Money with prayer for the issuance of a Temporary Restraining Order
and Writ of Preliminary Injunction against Optima. As a result, Optima was
constrained to file before the MeTC a Complaint for Unlawful Detainer and
Damages with Prayer for the Issuance of a TRO and/or Preliminary Mandatory
Injunction against Hertz. The MeTC rendered a Decision, ruling that petitioner
Optima had established its right to evict Hertz from the subject premises due
to nonpayment of rentals and the expiration of the period of lease. The RTC
affirmed it by dismissing the appeal in a Decision. Hertz thereafter filed a
verified Rule 42 Petition for Review on Certiorari with the CA. On appeal, the
CA ruled that, due to the improper service of summons, the MeTC failed to

acquire jurisdiction over the person of respondent Hertz. The appellate court
thereafter reversed the RTC and remanded the case to the MeTC to ensure
the proper service of summons.
ISSUE: Whether or not the MeTC properly acquired jurisdiction over the
person of respondent Hertz.
HELD: The MeTC acquired jurisdiction over the person of respondent Hertz. In
civil cases, jurisdiction over the person of the defendant may be acquired
either by service of summons or by the defendants voluntary appearance in
court and submission to its authority. In this case, the MeTC acquired
jurisdiction over the person of respondent Hertz by reason of the latters
voluntary appearance in court. Preliminarily, jurisdiction over the defendant
in a civil case is acquired either by the coercive power of legal processes
exerted over his person, or his voluntary appearance in court. In this case,
the records show that the following statement appeared in respondents
Motion for Leave to File Answer: In spite of the defective service of summons,
the defendant opted to file the instant Answer with Counterclaim with Leave
of Court, Hertz voluntarily appeared before the court, therefore by virtue of
the voluntary appearance of respondent Hertz before the MeTC, the trial court
acquired jurisdiction over respondent Hertz.

#2 Padlan v Dinglasan
G.R. No. 180321

March 20, 2013

Peralta, J.:

Facts:
Elenita Dinglasan was the registered owner of a parcel of land covered by TCT No. T-105602.
While on board a jeepney, Elenitas mother had a conversation with one Maura Passion regarding
the sale of the said property. Believing Maura to be a real estate agent, Elenitas mother gave the
TCT of the said land to Maura, which the latter subdivided and sold to different individuals
through different Deeds of Sale. A portion of the said land was sold to one Lorna Ong who had a
TCT issued in her name. A few months later, Lorna then sold the same land to petitioner Editha
Padlan for P4,000.
After learning of the fraud, respondents Dinglasan demanded that Padlan surrender the lot,
but the latter refused. They therefore filed a case before the RTC for the cancellation of Padlans
TCT. Summons was served to the petitioner through her mother Anita Padlan.
During trial, petitioner, through her counsel, filed an Opposition and claimed that the court
did not acquire jurisdiction over her because the summons was not validly served upon her, but
was done only by substituted service to her mother. She maintained that she has long been

residing in Japan after she married a Japanese national and that she only returns briefly in the
Philippines every two years.
The RTC rendered a decision in favor of the petitioner. The CA, however, rendered a decision
in favor of the respondents. Petitioner Padlan thus filed for Reconsideration, averring that the
lower court failed to acquire jurisdiction over both her and the subject matter of the case.

Issues:
WON the court acquired jurisdiction over the person of the petitioner.
WON the court acquired jurisdiction over the person of the subject matter.

Held:
NO. The court did not acquire jurisdiction over the petitioner as pursuant to Sec 15, Rule 14
of the Rules of Civil Procedure, when the defendant does not reside in the Philippines and the
subject of the action is property within the Philippines, service of summons may be effected
outside of the Philippines by personal service or by publication in a newspaper of general
circulation. Thus, the substituted service to the petitioners mother did not vest jurisdiction of the
petitioners person to the court.

NO. It is well-established that the assessed value of the real property subject of the
complaint must be alleged in order to determine the jurisdiction. In the instant case, only the
P4,000 sale price of the property was alleged, making it the sole basis for the value of the
property. Given the said price, it is clear that it is the MTC and not the RTC that has jurisdiction
over the case. Thus. the trial and the decision of the RTC is null and void for lack of jurisdiction
and the case was dismissed without prejudice.
3. Macasaet vs. Co (697SCRA187)
Benjamin, J:

G.R Nos. 156759


Date: 6/5/13

Facts: Co sued petitioners for an alleged libelous article published on Abante Tonite.
Regional trial court sheriff Raul Medina tried to effect the summons on the petitioners but failed
because they were out of the office. The sheriff returned to the office thrice within the same day
but the petitioners never returned to the office that day. He resorted to substituted service of
summons and explained that it was because they are always out and unavailable.
Petitioners waved for the dismissal of the complaint for the reason that the summons were not
served properly.
The regional trial court dismissed their motion and the petitioners filed their motion for
reconsideration but was again denied.

The petitioners brought their case to the court of appeals but it affirmed the decision of the lower
court and denied the motion to dismiss filed by the petitioners.
Issue: Whether the regional trial court acquired jurisdiction over the petitioners.
Held: Yes, petitioners insistence on personal service by the serving officer was demonstrably
superfluous.
The allegations of the defendants that the Sheriff immediately resorted to substituted service of
summons upon them when he was informed that they were not around to personally receive the
same is untenable. Because of that information and because of the nature of the work of the
defendants that they are always on field, so the sheriff resorted to substituted service of summons.
There was substantial compliance with the rules, considering the difficulty to serve the summons
personally to them because of the nature of their job which compels them to be always out and
unavailable. Additionally, They had actually received the summons served through their
substitutes, as borne out by their fillings of several pleadings in the regional trial court.
The sheriff cannot be expected to personally effect the summons at all times and may resort to
substitution.

4. Pat-Og vs Civil Service Commission


G.R. No. 198755, June 5, 2013
Mendoza, J.

Facts:
Pat-Og, a Public School teacher is a respondent in an administrative case. He
was found guilty for Simple Misconduct to which he was meted a penalty of
Six-Months suspension.
Pat-Og is now questioning his indictment since under the Magna Carta for
Teachers, he must be tried administratively by a committee to be constituted
pursuant to the said magna Carta.
Issue:
W/N the CSC validly acquired jurisdiction over the case of Pat-Og.
Ruling:
CSC validly acquired jurisdiction over the case of Pat-Og.
CSC practiced concurrent jurisdiction with the committee provided for under
the Magna Carta for Teachers. Moreover, under Article X-B of the Constitution,
the CSC has the Power to hear, try and decide cases of Public
Officers/Employees which includes Public School Teachers.

5. G.R. No. 174908


June 17, 2013
DARMA MASLAG, Petitioner,
vs.
ELIZABETH MONZON, WILLIAM GESTON, and REGISTRY OF DEEDS OF
BENGUET, Respondents.
DEL CASTILLO, J.:
JURISDICTION
Factual Antecedents:

In 1998, petitioner filed a Complaint for reconveyance of real property


with declaration of nullity of original certificate of title (OCT) against
respondents Elizabeth Monzon (Monzon), William Geston and the
Registry of Deeds of La Trinidad, Benguet. The Complaint was filed
before the Municipal Trial Court (MTC) of La Trinidad, Benguet.
After trial, the MTC found respondent Monzon guilty of fraud in
obtaining an OCT over petitioners property. It ordered her to reconvey
the said property to petitioner, and to pay damages and costs of suit.

Respondents appealed to the Regional Trial Court (RTC) of La Trinidad,


Benguet.

The RTC of La Trinidad, Benguet, Branch declared the MTC without


jurisdiction over petitioners cause of action. It further held that it will
take cognizance of the case pursuant to Section 8, Rule 40 of the Rules
of Court

On May 4, 2004, RTC Judge Diaz


Resolution reversing the MTC Decision.

Petitioner filed a Notice of Appeal from the RTCs May 4, 2004


Resolution.

Petitioner assailed the RTCs May 4, 2004 Resolution for reversing the
MTCs factual findings and prayed that the MTC Decision be adopted.

The CA dismissed petitioners appeal. It observed that the RTCs May 4,


2004 Resolution (the subject matter of the appeal before the CA) set
aside an MTC Judgment; hence, the proper remedy is a Petition for
Review under Rule 42, and not an ordinary appeal.

Petitioner sought reconsideration. She argued, for the first time, that
the RTC rendered its May 4, 2004 Resolution in its original jurisdiction.

De

Rivera

issued

She cited the earlier October 22, 2003 Order of the RTC declaring the
MTC without jurisdiction over the case.

The CA denied petitioners Motion for Reconsideration in its September


22, 2006 Resolution.

Issues:
Does the MTC have jurisdiction over the case involved?

Ruling:
Yes. Under the present state of the law, in cases involving title to real
property, original and exclusive jurisdiction belongs to either the RTC or the
MTC, depending on the assessed value of the subject property. Pertinent
provisions of Batas Pambansa Blg. (BP) 129, as amended by Republic Act (RA)
No. 7691, provides:
xxxx
SEC. 33. Jurisdiction of Metropolitan Trial Courts, Municipal Trial Courts and
Municipal Circuit Trial Courts in Civil Cases. Metropolitan Trial Courts,
Municipal Trial Courts and Municipal Circuit Trial Courts shall exercise:
xxxx
(3) Exclusive original jurisdiction in all civil actions which involve title to, or
possession of, real property, or any interest therein where the assessed value
of the property or interest therein does not exceed Twenty thousand pesos
(P20,000.00) or, in civil actions in Metro Manila, where such assessed value
does not exceed Fifty thousand pesos (P50,000.00) x x x.
In the case at bench, annexed to the Complaint is a Declaration of Real
Property dated November 12, 1991, which was later marked as petitioners
Exhibit "A", showing that the disputed property has an assessed value
ofP12,400 only. Such assessed value of the property is well within the
jurisdiction of the MTC. In fine, the RTC, thru Judge Cabato, erred in applying
Section 19(1) of BP 129 in determining which court has jurisdiction over the
case and in pronouncing that the MTC is divested of original and exclusive
jurisdiction.
As discussed above, the MTC has original and exclusive jurisdiction over the
subject matter of the case; hence, there is no other way the RTC could have
taken cognizance of the case and review the court a quos Judgment except
in the exercise of its appellate jurisdiction.

6. G.R. No. 185891


CASE: CATHAY PACIFIC V REYES (699 SCRA 725)
CASE DIGEST:
FACTS
In March 1997, respondent Wilfredo Reyes (Wilfredo) made a travel reservation with Sampaguita
Travel for his familys trip to Adelaide, Australia scheduled from 12 April 1997 to 4 May 1997.
Upon booking and confirmation of their flight schedule, Wilfredo paid for the airfare and was
issued four (4) Cathay Pacific round-trip airplane tickets for Manila-HongKong-AdelaideHongKong-Manila. The family then were able to fly to Adelaide, Australia without a hitch. One
week before they were scheduled to fly back home, Wilfredo reconfirmed his familys return flight
with Cathay Pacific office and they were advised that the reservation was still okay as scheduled.
It was then when the family arrived at the airport knew that that their itinerary did not include
booking for the return flights except for Sixta Lapuz (mother-in-law). After a series of negotiation
with Cathay Pacific personnel, the family was accommodated and transported back to Manila
then eventually led to filling of this present case for the payment of damages it caused them.

ISSUE
1. Whether or not there is a valid cause of action against Cathay Pacific and Sampaguita Travel
filed by Wilfredo Reyes, Juanita Reyes, Michael Roy Reyes, and Sixta Lapuz.
RULING
Respondents cause of action against Cathay Pacific stemmed from a breach of contract of
carriage wherein said respondents entered into a contract of carriage with Cathay Pacific. The
passengers were holding valid and confirmed airplane tickets. Wilfredo was even assured by a
staff of Cathay Pacific that he does not need to reconfirm his booking.
Article 1370 of the Civil Code mandates that "if the terms of a contract are clear and leave no
doubt upon the intention of the contracting parties, the literal meaning of its stipulations shall
control." Under Section 9, Rule 130 of the Rules of Court, once the terms of an agreement have
been reduced to writing, it is deemed to contain all the terms agreed upon by the parties and no
evidence of such terms other than the contents of the written agreement shall be admissible. The
terms of the agreement of appellants and appellee Cathay Pacific embodied in the tickets issued
by the latter to the former are plain Cathay Pacific will transport the Reyeses to Adelaide,
Australia from Manila and back to Manila via Hongkong. In addition, the tickets reveal that all
appellants have confirmed bookings for their travel. When appellee Cathay Pacific initially
refused to transport appellants to the Philippines on 4 May 1997 due to the latters lack of
reservation, it has, in effect, breached their contract of carriage. Appellants, however, were
eventually accommodated and transported by appellee Cathay Pacific to Manila.
Since the contract between the parties is an ordinary one for services, the standard of care
required of respondent is that of a good father of a family under Article 1173 of the Civil Code. The
court ruled that Sampaguita Travel failed to exercise due diligence in performing its obligations
under the contract of services. Thus, Cathay Pacific and Sampaguita Travel acted together in
creating the confusion in the bookings which led to the erroneous cancellation of respondents
bookings. Their negligence is the proximate cause of the technical injury sustained by
respondents. Therefore, they have become joint tortfeasors, whose responsibility for quasi-delict,
under Article 2194 of the Civil Code, is solidary. Cathay Pacific and Sampaguita Travel are jointly

and solidarily liable for nominal damages worth P25,000.00 awarded to respondents Wilfredo,
Juanita and Michael Roy.
On the other hand, the complaint of respondent Sixta Lapuz is DISMISSED for lack of cause of
action against Cathay Pacific or Sampaguita Travel. In order to consider, the following elements
of cause of action must be present: (1) a right existing in favor of the plaintiff, (2) a duty on the
part of the defendant to respect the plaintiffs right, and (3) an act or omission of the defendant in
violation of such right. But in the absence of such, no violation of any right or breach of any duty
on the part of Cathay Pacific and Sampaguita Travel to Sixta because she is a holder of a valid
booking. Hence, Sixta is deprived of any relief against either Cathay Pacific or Sampaguita Travel.
7. Anchor Savings, Inc. Vs Furigay
G.R. No. 191178

March 13, 2013

Mendoza, J.
Facts:
On April 21, 1999, ASB filed a verified complaint for sum of money and damages with application
for replevin against Ciudad Transport Services, Inc. (CTS), its president, respondent Henry H.
Furigay; his wife, respondent Gelinda C. Furigay; and a John Doe. The case was docketed as
Civil Case No. 99-865 and raffled to Branch 143 of the Regional Trial Court of Makati City (RTC).
While Civil Case No. 99-865 was pending, respondent spouses donated their registered properties
in Alaminos, Pangasinan, to their minor children, respondents Hegem G. Furigay and Herriette C.
Furigay. As a result, Transfer Certificate of Title (TCT) Nos. 21743,7 21742,8 21741,9 and 2174010
were issued in the names of Hegem and Herriette Furigay.
Claiming that the donation of these properties was made in fraud of creditors, ASB filed a
Complaint for Rescission of Deed of Donation, Title and Damages11 against the respondent
spouses and their children.
Issue:
Whether or Not Anchor Savings Bank has a cause of action againt Sps. Furigay.
Ruling:
No. Spouses Furigay have no cause of action against Sps. Furigay.
Section 1 of Rule 2 of the Revised Rules of Court requires that every ordinary civil action must be
based on a cause of action. Section 2 of the same rule defines a cause of action as an act or
omission by which a party violates the right of another. In order that one may claim to have a
cause of action, the following elements must concur: (1) a right in favor of the plaintiff by
whatever means and under whatever law it arises or is created; (2) an obligation on the part of the
named defendant to respect or not to violate such right; and (3) an act or omission on the part of
such defendant in violation of the right of the plaintiff or constituting a breach of the obligation of
the defendant to the plaintiff for which the latter may maintain an action for recovery of damages
or other appropriate relief

In relation to an action for rescission, it should be noted that the remedy of rescission is
subsidiary in nature; it cannot be instituted except when the party suffering damage has no other
legal means to obtain reparation for the same.
A cursory reading of the allegations of ASBs complaint would show that it failed to allege the
ultimate facts constituting its cause of action and the prerequisites that must be complied before
the same may be instituted. ASB, without availing of the first and second remedies, that is,
exhausting the properties of CTS, Henry H. Furigay and Genilda C. Furigay or their transmissible
rights and actions, simply undertook the third measure and filed an action for annulment of the
donation. This cannot be done.
Thus, ASB has no cause of action against Sps. Furigay with regard to the rescission of the
donation.
8. 700 SCRA 778
Heirs of Ypon vs. Ricaforte
CAUSE OF ACTION
FACTS:
On July 29, 2010, petitioners, together with some of their cousins, 4 filed a complaint for
Cancellation of Title and Reconveyance with Damages (subject complaint) against respondent
Gaudioso Ponteras Ricaforte a.k.a. "Gaudioso E. Ypon" (Gaudioso). In their complaint, they
alleged that Magdaleno Ypon (Magdaleno) died intestate and childless on June 28, 1968, leaving
behind Lot Nos. 2-AA, 2-C, 2-F, and 2-J which were then covered by Transfer Certificates of Title
(TCT) Nos. T-44 and T-77-A. 6 Claiming to be the sole heir of Magdaleno, Gaudioso executed an
Affidavit of Self-Adjudication and caused the cancellation of the aforementioned certificates of
title, leading to their subsequent transfer in his name under TCT Nos. T-2637 and T-2638, 7 to the
prejudice of petitioners who are Magdalenos collateral relatives and successors-in-interest. 8
In his Answer, Gaudioso alleged that he is the lawful son of Magdaleno as evidenced by: (a) his
certificate of Live Birth; (b) two (2) letters from Polytechnic School; and (c) a certified true copy of
his passport.9 Further, by way of affirmative defense, he claimed, among others that: (a)
petitioners have no cause of action against him.
ISSUE: W/N the petitioners have cause of action against the defendant
Ruling:
NO.
Cause of action is defined as the act or omission by which a party violates a right of another. 16 It is
well-settled that the existence of a cause of action is determined by the allegations in the
complaint.17 In this relation, a complaint is said to assert a sufficient cause of action if, admitting
what appears solely on its face to be correct, the plaintiff would be entitled to the relief prayed
for.18Accordingly, if the allegations furnish sufficient basis by which the complaint can be
maintained, the same should not be dismissed, regardless of the defenses that may be averred by
the defendants.

As stated in the subject complaint, petitioners, who were among the plaintiffs therein, alleged that
they are the lawful heirs of Magdaleno and based on the same, prayed that the Affidavit of SelfAdjudication executed by Gaudioso be declared null and void and that the transfer certificates of
title issued in the latters favor be cancelled. While the foregoing allegations, if admitted to be
true, would consequently warrant the reliefs sought for in the said complaint, the rule that the
determination of a decedents lawful heirs should be made in the corresponding special
proceeding20 precludes the RTC, in an ordinary action for cancellation of title and reconveyance,
from granting the same.
Jurisprudence dictates that the determination of who are the legal heirs of the deceased must be
made in the proper special proceedings in court, and not in an ordinary suit for recovery of
ownership and possession of property. This must take precedence over the action for recovery of
possession and ownership.
9. G.R. No. 141809
CASE: INGLES v. ESTRADA (695 SCRA 285)
CASE DIGEST:
FACTS:
On 14 April 1993, petitioner spouses Jose and Josefina Ingles obtained a loan from respondent
Charles Esteban. As collateral for such loan, petitioners mortgaged their property in favor of
Charles.
On 29 May 1993, the loan matured and petitioners issued a check unfortunately, the check
bounced. On 30 October 1993, Jose died and was survived by Josefina and their children. On 13
July 1994, Charles sent a letter demanding for the payment of the loan yet still Josefina failed to
pay. This causes respondent Charles petition for the extrajudicial foreclosure of the said property
which was later ordered by the respondent judge directing the clerk of court and ex officio sheriff
to proceed with the extrajudicial foreclosure.
ISSUE:
Whether or not the CA erred in dismissing Ingleses petition for Annulment of Final Orders.
HELD:
NO. The subject of the Ingleses petition for Annulment of Final Orders are not proper subjects of
a petition for annulment before the Court of Appeals. The assailed orders of Executive Judge
Estrada are not the final orders in civil actions of the RTC that may be subject of annulment by
the CA under Rule 47. There is a clear-cut difference between issuances made in a civil action on
one hand and orders rendered in a proceeding for the extrajudicial foreclosure of a mortgage on
the other.
Civil Actions are suits filed in court involving either the enforcement or protection of a right, or
the prevention or redress of a wrong. They are commenced by the filing of an original complaint
before ane appropriate court and their proceedings are governed by the provisions of the Rules of
Court on ordinary or special civil actions. Civil Actions are adversarial in nature, presupposing the
existence of disputes defined by the parties that are, in turn, submitted before the court for
disposition. Issuances made therein, including and most especially judgments, final orders or

resolutions, are therefore rendered by courts in the exercise of their judicial function. Proceedings
for the extrajudicial foreclosure of mortgages are not adversarial, as the judge merely performs
therein an administrative function to ensure that all requirements for the extrajudicial foreclosure
of a mortage are satisfied. Thus, in First Marbella Condominium Assoc., Inc. V. Gatmaytan theee
court ruled that the orders of the executive judge in such proceedings, whether they be to allow or
disallow the extrajudicial forclosure are not issued in the exercise of a judicial function but issued
in the exercise of administrative function to supervise the ministerial duty of the Clerk of Court as
Ex Officio Sheriff.
#10 Yu v. Pacleb
G.R. No. 172172, February 24, 2009
PUNO, C.J.

Facts:
Respondent Baltazar Pacleb and his wife, Angelita Chan, are registered owners of a parcel of land
located in Dasmarinas, Cavite (Langcaan Property). In 1992, the Langcaan Property became the
subject of 3 documents purporting to transfer its ownership. First was a Deed of Absolute Sale
between Spouses Pacleb and Angelita Chan and Rebecca Del Rosario. Second was a Deed of
Absolute Sale between Del Rosario and Ruperto Javier. And third was a Contract to Sell between
Javier and Spouses Yu. All the aforementioned sales were not registered. Subsequently, spouses
Yu filed with the RTC a complaint for specific performance and damages against Javier to compel
him to deliver to them ownership and possession, as well as title to the Langcaan Property. They
alleged that after giving their initial payment they discovered that the property was tenanted by
Ramon Pacleb. In 1994, RTC rendered judgment in favor of the spouses. The decision became
final. Later, respondent filed a complaint for annulment of deed of sale purportedly executed
between him and his wife and Rebecca Del Rosario alleging that their signatures thereon were
forgeries. Respondent, later, moved to dismiss the case, which the court granted. Meanwhile,
spouses Yu filed an action with the MTC for forcible entry against respondent, alleging that they
had prior physical possession of the Langcaan Property. MTC ruled in favor of spouses Yu, which
decision was affirmed by the RTC. However, the CA set aside the decisions and found that it was
respondent who had prior physical possession of the property. Subsequently, respondent filed the
instant case for removal of cloud from title with damages, which the RTC dismissed holding that
spouses Yu are purchasers in good faith. However, the CA set aside said decision and ruled that
the 1994 decision of the RTC which has became final, did not transfer ownership of the Langcaan
Property to the spouses. MR denied.

Issue:
WON ownership over the Langcaan Property was properly vested in petitioner spouses by virtue
of the 1994 decision.

Held: No. Such case under which that decision was rendered is an action for specific performance
and damages filed by petitioner spouses against Javier to compel performance of the latters

undertakings under their Contract to Sell. As correctly held by the Court of Appeals, its object is to
compel Javier to accept the full payment of the purchase price, and to execute a deed of absolute
sale over the Langcaan Property in their favor. The obligations of Javier under the contract to sell
attach to him alone, and do not burden the Langcaan Property. The Court have held in an
unbroken string of cases that an action for specific performance is an action in personam. In
Cabutihan v. Landcenter Construction and Development Corporation, it was ruled
that an action for specific performance praying for the execution of a deed of sale in connection
with an undertaking in a contract, such as the contract to sell, in this instance, is an action in
personam. Being a judgment in personam, the case is binding only upon the parties properly
impleaded therein and duly heard or given an opportunity to be heard. Therefore, it cannot bind
respondent since he was not a party therein. Neither can respondent be considered as privy
thereto since his signature and that of his late first wife, Angelita Chan, were forged in the deed of
sale.
11. CSC vs Almojuela
G.R. No. 194368
BRION, J.

April 2, 2013

FACTS:
An administrative case was filed against Desk Officer/ Supervisor SJO2 Almojuela, because a
detention prisoner, Tony Lao escaped in the Makati City Jail. A BJMP Investigation Report
conducted on the incident concluded that SJO2 Almojuela and the rest of the jail officers on third
shift custodial duty all colluded to facilitate Laos getaway. SJO2 Almojuela and JO1 Loyola were
found guilty of Grave Misconduct and were meted the penalty of dismissal from the service. SJO2
Almojuela and JO1 Loyola moved for the reconsideration of Director Walits decision, which the
latter denied for lack of merit in a Joint Resolution dated June 21, 2006. SJO2 Almojuela then
appealed his conviction before the Civil Service Commission (CSC), which affirmed Director
Walits decision in its Resolution No. 080701. The CSC subsequently denied SJO2 Almojuelas
motion for reconsideration. SJO2 Almojuelas next recourse was a petition for review before the
Court of Appeals. SJO2 Almojuela claimed to have been denied due process because he was not
accorded the benefit of a full-blown trial. Almojuela filed a motion for reconsideration. The
appellate court partially granted SJO2 Almojuelas motion for reconsideration, and lowered his
liability from grave to simple misconduct. Applying Section 54(b), Rule IV of the Uniform Rules
on Administrative Cases in Civil Service,36 SJO2 Almojuela was meted the penalty of three
months suspension as there was neither any attendant mitigating nor aggravating circumstance.
The CA found no corrupt motive or willful intent on SJO2 Almojuelas part to violate the BJMP
Rules and Regulations. No clear evidence was presented to show that SJO2 Almojuela was
directly involved in the prison break, nor was it proven that he benefited from it. SJO2 Almojuela
likewise did not willfully trifle with the BJMP Rules and Regulations. Hence, this present petition.
The CSC asserts in its present petition that the CA should not have had disturbed the CSCs
findings, as conclusions of administrative bodies charged with their specific field of expertise are
generally afforded great weight by the courts. SJO2 Almojuelas conviction is supported by
evidence on record, and sufficiently satisfied the substantial evidence standard. Taken together,
the testimonies submitted during the BJMP investigation establish that SJO2 Almojuela connived
with JO1 Pascual, JO1 Loyola and Lao to facilitate the latters escape. Even assuming that SJO2
Almojuela had no knowledge of the plan, he could have easily discovered and prevented the
escape had he been awake and alert.
ISSUE:
Whether the CSCs petition for review on certiorari should be dismissed for failure to comply with
Section 4, Rule 45 of the Rules of Court;
HELD:

YES. The consequences of this mistep are prejudicial to the party filing the pleading. Section 5,
Rule 45 of the Rules of Court provides that a petition for review that does not comply with the
required certification against forum shopping is a ground for its dismissal. This certification must
be executed by the petitioner, not by counsel. It is the petitioner, and not always the counsel
whose professional services have been retained only for a particular case, who is in the best
position to know whether he or it actually filed or caused the filing of a petition in that case.
Hence, a certification against forum shopping by counsel is a defective certification. It is
equivalent to non-compliance with the requirement under Section 4, Rule 45 and constitutes a
valid cause for dismissal of the petition. However, there have been instances when the demands of
substantial justice convinced us to apply the Rules liberally by way of compliance with the
certification against forum shopping requirement; the rule on certification against forum
shopping, while obligatory, is not jurisdictional. Justifiable cirsumtances may intervene and be
recognized, leading the Court to relax the application of this rule
12. Reyes vs. NLRC

G.R. No. 180551


Facts:
Before this Court is a Special Civil Action for Certiorari under Rule 65 of the
Revised Rules of Court filed by petitioner Erwin H. Reyes seeking to reverse
and set aside the Resolutions the Court of Appeals. the appellate court
dismissed petitioners Petition for Certiorari therein for failure to give an
explanation why copy of the said Petition was not personally served upon the
counsel of the respondents.
The Petitioner file a Complaint for illegal dismissal with claims for moral and
exemplary damages and attorneys fees against the respondents Coca Cola
Bottlers Philippines (CCBP) and Rotaida Taguibao (Taguibao) before the Labor
Arbiter on 14 June 2004. In his Complaint, petitioner alleged that he was first
employed by respondent CCBP, through Interserve Manpower Agency
(Interserve), as a Leadman. Petitioner was initially assigned to the Mendiola
Sales Office of respondent CCBP. Petitioners employment contract was
renewed every five months and he was assigned a different task every
time. Such an arrangement continued until petitioner was directly hired by
respondent CCBP as a Route Salesman. Exactly one year from the time of
petitioners employment as a Route Salesman, respondent CCBP, thru
Taguibao, terminated his services where he already acquired the status of a
regular employee, petitioner asserted that his dismissal from employment
without the benefit of due process was unlawful. opposing the Complaint,
respondent CCBP refuted petitioners allegation that he was a regular
employee. Petitioners employment was for a fixed period of three months,
which was subsequently extended with petitioners consent. the Labor Arbiter
promulgated his Decision, favoring petitioner, since there was insufficient
evidence to sustain the averment of respondents CCBP and Taguibao that
petitioners employment was for a fixed period. the respondents reinstated
the petitioner to his former position. However, respondents CCBP and
Taguibao, Appealed before the NLRC but later on dismissed. NLRC reduced
the amount of backwages awarded to petitioner and deleted the order for his
reinstatement. All the parties moved for the reconsideration of the foregoing
NLRC Decision but denied. The petitioner elevated his case before the Court
of Appeals by filing a Petition for Certiorari he averred in his Petition that the

NLRC abused its discretion in ignoring the established facts and legal
principles when it modified the award for his backwages and deleted the
order for his reinstatement.
The Court of Appeals dismissed petitioners Petition for Certiorari for his
failure to give any explanation why a copy of the said Petition was not
personally served upon the counsel of the adverse parties. Since petitioner
failed to timely file a Motion for Reconsideration, the Resolution dated 10
November 2006 of the Court of Appeals became final and executory, and an
Entry of Judgment was made.
On 19 July 2007, petitioners new counsel filed an Entry of Appearance
with an Urgent Motion for Reconsideration. Petitioner, through his new
counsel, sought for the liberality of the Court of Appeals, faulting his former
counsel for the procedural defects of his Petition and for his failure to
seasonably seek reconsideration of the 10 November 2006 Resolution of the
appellate court. Also, this time, it would appear that petitioner provided the
explanation required by Section 11, Rule 13 of the Revised Rules of Court. In a
Resolution dated 9 November 2007, the Court of Appeals denied petitioners
Urgent Motion for Reconsideration for being filed out of time.
Hence,
petitioner
via
the
instant
Special
Civil
Action
for Certiorari assails the Resolutions of the Court of Appeals to the Supreme
Court.
Issue
Whether or not the for petitioners failure to comply with Section 11, Rule 13
of the Revised Rules of Court, should expunged his case from the records
even though there is a negligence on the part of his former counsel?
Held
It is true that for petitioners failure to comply with Section 11, Rule 13 of the
Revised Rules of Court, his petition should be expunged from the records.
Nevertheless, the Rules of Court itself calls for its liberal construction, with
the view of promoting their objective of securing a just, speedy and
inexpensive disposition of every action and proceeding. The Court is fully
aware that procedural rules are not to be belittled or simply disregarded for
these prescribed procedures insure an orderly and speedy administration of
justice. However, it is equally true that litigation is not merely a game of
technicalities. Law and jurisprudence grant to courts the prerogative to relax
compliance with procedural rules of even the most mandatory character,
mindful of the duty to reconcile both the need to put an end to litigation
speedily and the parties right to an opportunity to be heard
In numerous cases,[12] the Court has allowed liberal construction of
Section 11, Rule 13 of the Revised Rules of Court when doing so would be in
the service of the demands of substantial justice and in the exercise of the
equity jurisdiction of this Court. In one such case, Fulgencio v. National Labor

Relations Commission,[13] this Court provided the following justification for its
non-insistence on a written explanation as required by Section 11, Rule 13 of
the Revised Rules of Court:
The rules of procedure are merely tools designed to
facilitate the attainment of justice. They were conceived
and promulgated to effectively aid the court in the
dispensation of justice.Courts are not slaves to or robots
of technical rules, shorn of judicial discretion. In rendering
justice, courts have always been, as they ought to be,
conscientiously guided by the norm that on the balance,
technicalities take a backseat against substantive rights, and not
the other way around. Thus, if the application of the Rules would
tend to frustrate rather than promote justice, it is always within
our power to suspend the rules, or except a particular case from
its operation.
The call for a liberal interpretation of the Rules is even more strident in
the instant case which petitioners former counsel was obviously negligent in
handling his case before the Court of Appeals. It was petitioners former
counsel who failed to attach the required explanation to the Petition in. Said
counsel did not bother to inform petitioner, his client, of the Resolution of the
appellate court dismissing the Petition for lack of the required
explanation. Worse, said counsel totally abandoned petitioners case by
merely allowing the reglementary period for filing a Motion for
Reconsideration to lapse without taking any remedial steps; thus, the 10
November 2006 Resolution became final and executory.
The basic general rule is that the negligence of counsel binds the
client. Hence, if counsel commits a mistake in the course of litigation, thereby
resulting in his losing the case, his client must perforce suffer the
consequences of the mistake. The reason for the rule is to avoid the
possibility that every losing party would raise the issue of negligence of his or
her counsel to escape an adverse decision of the court, to the detriment of
our justice system, as no party would ever accept a losing verdict. This
general rule, however, pertains only to simple negligence of the
lawyer. Where the negligence of counsel is one that is so gross,
palpable, pervasive, reckless and inexcusable, then it does not bind
the client since, in such a case, the client is effectively deprived of
his or her day in court.[14]
The circumstances of this case qualify it under the exception, rather
than the general rule. The negligence of petitioners former counsel may be
considered gross since it invariably resulted to the foreclosure of remedies
otherwise readily available to the petitioner. Not only was petitioner deprived
of the opportunity to bring his case before the Court of Appeals with the
outright dismissal of his Petition on a technicality, but he was also robbed of
the chance to seek reconsideration of the dismissal of his Petition. What
further impel this Court to heed the call for substantial justice are the
pressing merits of this case which, if left overshadowed by technicalities,

could result in flagrant violations of the provisions of the Labor Code and of
the categorical mandate of the Constitution affording protection to labor.
13. Summa Kumagai vs. Romago Inc
G.R. No. 177210 April 7, 2009
PONENTE: Justice Chico-Nazario, J.
FACTS:
Summa Kumagai Kumagai Gumi (SK-KG) engaged the services of Romago Inc. (RI) under a SubContract Agreement for the electrical works needed in the construction of the superstructure of a
medical center. A dispute arose due to a refusal on both sides to fulfill their obligations. During
the hearings, when RI tried to present evidence to controvert the counterclaims of SK-KG, the
Arbitration Panel did not allow RI to do so on the ground that its failure to file a reply to the
answer was deemed an admission of SK-KGs counterclaims. RI also filed a motion to submit
additional evidence but it was denied by the Construction Industry Arbitration Commission
(CIAC).
ISSUE:
Was the ruling by the CIAC correct?
RULING:
No. The CIAC is mistaken in denying the attempt of RI to present evidence against the
counterclaims of the SK-KG on the ground that its failure to reply to SK-KGs answer was already
an admission of the counterclaims in said answer. There is no basis for such a conclusion.
Even under the Rules of Court (Rule 6, Section 10) which are stricter than those of quasi-judicial
bodies, if a party fails to file such reply, all the new matters alleged in the answer are deemed
controverted. If, in administrative proceedings, these technical rules of procedure and evidence
are not supposed to be strictly applied and administrative due process should not be fully equated
with due process in its strict judicial sense, then it is completely unreasonable for an
administrative body like the CIAC to be even more severe than the courts when it comes to the
filing of a reply. To be sure, CIAC rules even explicitly direct them to use every reasonable means
to ascertain the facts in each case speedily and objectively without regard to technicalities, all in
the interest of substantive due process. The CIAC should not have taken the evidence of SK-KG
hook line and sinker and should have used all means to ascertain all the facts in interest of
substantial justice. There was a violation of RIs right to due process.
It does well for the CIAC Arbitrators to remember that the CIAC Rules explicitly direct them to
use every and all reasonable means to ascertain the facts in each case speedily and objectively
without regard to technicalities of law and procedure, all in the interest of
substantive due process.

14. De Grano vs. Lacaba


589 SCRA 148
FACTS:
Respondent Gregorio Lacaba claims that he is the owner of two adjacent parcels of land in Laurel,
Batangas covered by a separate tax declaration under his name. Respondent Lacaba further

alleges that sometime in the second week of May 2000, the petitioner by means of force,
intimidation, strategy and threats destroyed the perimeter fence of the property built by the
respondent. On May 30, 2000, the respondent filed a complaint for forcible entry with prayer for
a Temporary Restraining Order and/or Preliminary injuction against petitioner de Grano. The
respondent further claims that he has been in physical possession of the two parcels of land for
more than 30 years and has been paying the real property taxes for the two properties. The
petitioner in his answer averred that the real owners and possessors of the property were the
family of Ernesto Malabanan as evidenced by TCT No. T-31929 of the Register of Deeds of
Tanauan, Batangas. Other documents proving the same were also submitted. The Municipal
Circuit Trial Court dismissed the complaint for lack of cause of action on August 11, 2000. RTC
affirmed the decision of MCTC on November 13, 2000 from which the respondent filed a motion
for reconsideration. RTC denied the motion for reconsideration in an order dated March 28,
2001. A copy of the resolution was received by respondents counsel on April 18, 2001. On
October 23, 2001, the petitioner manifested that it was not he who filed the motion for
reconsideration and hence, the RTC modified its March 28, 2001 changing defendant to
plaintiff. The respondent then filed a motion for extension of time to file a petition for review
with the CA on November 27, 2001. The respondent based the filing of this motion on the October
23rd order of the RTC. On December 12. 2001 the respondent filed a petition for review with the
CA. Petitioner then filed a manifestation with motion to dismiss the instant petition and to cite
the petitioners (herein repondents) and the petitioners counsel for contempt. The petitioner
alleged that the respondent deliberately concealed the fact that the petition was filed out of time
by not attaching the March 28, 2001 RTC order which denied the respondents motion for
reconsideration. On October 16, 2002, a decision was rendered by the CA reversing and setting
aside the decision of the RTC on November 13, 2000 and October 23, 2001 denying the motion
for reconsideration of the said decision. The petitioner filed a motion for reconsideration and later
filed a Manifestation with Request for Judicial Notice of the Verification survey conducted by the
DENR on February 15, 2002 which shows the property was part of the parcel of land registered
under the Malabanan Family. The CA denied the Motion for reconsideration of the petitioner on
June 18, 2003, Hence, this petition for review.
ISSUE:
Whether or not the Court of Appeals gravely erred in taking cognizance and/or giving due course
to the petition for review filed before it by respondent Lacaba.
RULING:
Yes. The CA erred in taking cognizance of the petition for review that was filed way beyond the
reglementary period. The Supreme Court ruled that the Rules of procedure may be relaxed in the
interest of substantial justice and in order to give litigant the fullest opportunity to establish the
merits of his complaint. However, concomitant to a liberal application of the rules of procedure
should be an effort on the part of the party invoking liberality to explain its failure to comply with
the rules. And prove the existence of exceptionally meritorious circumstances warranting such
liberality.
The reglementary period should have been counted from the receipt of the March 28, 2001 order
and not the October 28, 2001 RTC order as what the respondent have done as a stratagem or out
of plain ignorance. Further, the delay was not explained and the respondent also failed to
acknowledge that he filed a petition for review with the CA beyond the prescriptive period.
The Supreme Court ruled that when a judgment is amended, the date of the amendment should
be considered the date of the decision in the computation of the period for perfecting the appeal.
However, this rule presupposes that the amendment consists of a material alteration of such
substance and proportion that would, in effect, give rise to an entirely new judgment. But when
the amendment merely consists of the correction of a clerical error, no new judgment arises. In

such case, the period of filing the appeal should be counted from the receipt of the original
judgment.
Since no material alteration was present in the judgment, for the amendment only consists of the
changing of the word defendant with plaintiff in the dispositive portion, the prescriptive
period must be counted from date of the receipt of the original order which was April 18, 2001.
The respondent had until May 3, 2001 to file a petition for review however he filed a motion for
extension only on November 27, 2001. Delay for almost 7 months cannot be considered
reasonable by the Court.

15. Navarro vs. Escobido

G.R. No. 153788

November 27, 2009

Brion, J.
Facts:
Karen Go filed two complaints to the RTC for replevin and sum of money with
damages against Navarro in order to recover two vehicles in the latters
posession. However, Navarro, in his answer raised the defense that Karen Go
has no cause of action and alleged that Go is not a real party-in-interest
because she was not a signatory to the agreement between Glenn Go
(Husband of Karen Go) and Navarro with regard to the vehicles.
Initially, RTC dismissed the case but later, reversed it self on the ground that,
Glenn Gos car leasing business is a conjugal property. Thus, the court
ordered Karen Go to file a motion to include Glenn Go as a co-plaintiff to the
case.
Navarro then filed a Petition for Certiorari before the CA. According to
Navarro, a complaint which failed to state a cause of action could not be
converted into one with a cause of action by mere amendment or
supplemental pleading. CA denied petition.
Issue:
Whether or Not Karen Go is a real party in interest ?
Ruling:
Yes, because Karen Go is the registered owner of Kargo Enterprises.
Glenn and Karen Go are effectively co-owners of Kargo Enterprises and the
properties registered under this name; hence, both have an equal right to
seek possession of these properties. Therefore, only one of the co-owners,
namely the co-owner who filed the suit for the recovery of the co-owned

property, is an indispensable party thereto. The other co-owners are not


indispensable parties. They are not even necessary parties, for a complete
relief can be accorded in the suit even without their participation, since the
suit is presumed to have been filed for the benefit of all co-owners.
Moreover, Section 4 of Rule 3 states that, Husband and Wife shall sue or be
sued jointly, except as provided by law
Thus, Karen Go is a real party in interest.
16. REBECCA PACAA-CONTRERAS and ROSALIE
PACAA, Petitioners, vs. ROVILA WATER SUPPLY, INC., EARL U
KOKSENG, LILIA TORRES, DALLA P. ROMANILLOS and MARISSA
GABUYA, Respondents.
G.R. No. 168979
December 2, 2013
BRION, J
Facts:
The petitioners claimed that their family has long been known in the
community to be engaged in the water supply business; they operated the
"Rovila Water Supply". The petitioners alleged that Lilia was a former trusted
employee in the family business who hid business records and burned and
ransacked the family files.
She then claimed ownership over the family business through a
corporation named "Rovila Water Supply, Inc." (Rovila Inc.) Upon inquiry with
the Securities and Exchange Commission (SEC), the petitioners claimed that
Rovila Inc. was surreptitiously formed with the respondents as the majority
stockholders. The respondents did so by conspiring with one another and
forming the respondent corporation to takeover and illegally usurp the family
business registered name.
In forming the respondent corporation, the respondents allegedly used
the name of Lourdes as one of the incorporators and made it appear in the
SEC documents that the family business was operated in a place other than
the Pacaa residence. Thereafter, the respondents used the Pacaa familys
receipts and the deliveries and sales were made to appear as those of the
respondent Rovila Inc. Using this scheme, the respondents fraudulently
appropriated the collections and payments.
The petitioners filed the complaint in their own names although Rosalie
was authorized by Lourdes through a sworn declaration and special power of
attorney (SPA). The respondents filed a first motion to dismiss on the ground
that the RTC had no jurisdiction over an intra-corporate controversy.
The RTC denied the motion. On September 26, 2000, Lourdes died and
the petitioners amended their complaint, with leave of court, on October 2,
2000 to reflect this development. They still attached to their amended

complaint the sworn declaration with SPA, but the caption of the amended
complaint remained the same.
Issue:
Whether or not the petitioners are the real parties in interest?
Ruling:
Yes. The heirs of the spouses Luciano and Lourdes Pacaa, except
herein petitioner and Lagrimas Pacaa-Gonzalez, are ORDERED IMPLEADED
as parties plaintiffs and the RTC is directed tp proceed with the trial of the
case with DISPATCH.
It should be emphasized that insofar as the petitioners are concerned,
the respondents have waived the dismissal of the complaint based on the
ground of failure to state a cause of action because the petitioners are not
the real parties in interest. At this juncture, a distinction between a real party
in interest and an indispensable party is in order. In Carandang v. Heirs of de
Guzman, et al.,57 the Court clarified these two concepts and held that "[a]
real party in interest is the party who stands to be benefited or injured by the
judgment of the suit, or the party entitled to the avails of the suit. On the
other hand, an indispensable party is a party in interest without whom no
final determination can be had of an action, in contrast to a necessary party,
which is one who is not indispensable but who ought to be joined as a party if
complete relief is to be accorded as to those already parties, or for a
complete determination or settlement of the claim subject of the action. xxx
If a suit is not brought in the name of or against the real party in interest, a
motion to dismiss may be filed on the ground that the complaint states no
cause of action. However, the dismissal on this ground entails an examination
of whether the parties presently pleaded are interested in the outcome of the
litigation, and not whether all persons interested in such outcome are actually
pleaded. The latter query is relevant in discussions concerning indispensable
and necessary parties, but not in discussions concerning real parties in
interest. Both indispensable and necessary parties are considered as real
parties in interest, since both classes of parties stand to be benefited or
injured by the judgment of the suit."
At the inception of the present case, both the spouses Pacaa were not
impleaded as parties-plaintiffs. The Court notes, however, that they are
indispensable parties to the case as the alleged owners of Rovila Water
Supply. Without their inclusion as parties, there can be no final determination
of the present case. They possess such an interest in the controversy that a
final decree would necessarily affect their rights, so that the courts cannot
proceed without their presence. Their interest in the subject matter of the suit
and in the relief sought is inextricably intertwined with that of the other
parties.
17. OCO VS. LIMBARING
G.R. No. 161298 January 31, 2006

PANGANIBAN, CJ
FACTS:
Sabas Limbaring subdivided his Lot 2325-D into two lots. He executed Deeds of Sale in favor of
Jennifer Limbaring and Sarah Jane Limbaring. After said transaction, Percita Oco, a daughter of
Sabas Limbaring, went to Ozamis City because she sensed some irregularities in said transaction.
Oco then filed a case for perjury and falsification of documents against respondent , who is the
father of Jennifer and Sarah Jane.
Eventually, it was decided that subject subdivided lots be reconveyed to Oco. In return, oco shall
pay respondent all the expenses amounting to PhP 25,000 that had been incurred during the
transfer of the title. After the reconveyance, Oco left Ozamis City without paying the agreed
amount to respondent.
Respondent then filed a complaint against spouses Oco for the recission of the sales contracts,
with recovery of possession and ownership of the two parcels of land. In their defense, spouses
Oco claimed that respondent was not a real party in interest given that he was not the one whom
they entered the contracts with and prayed for the Dismissal of the Complaint. However,
respondent insisted that he was the actual buyer of the lots, and contended that he was a trustor,
whose property was being held in trust by his daughters.
The Complaint was dismissed by the Regional Trial Court on the ground that respondent was not
a real party in interest. This Decision was reversed by the Court of Appeals. The Court of Appeals
held that a trust relationship was created when respondent purchased the lots in favor of his
daughters. Therefore, he is a real party in interest.
ISSUE:
Whether or not respondent is a real party in interest.
HELD:
It was held by the Supreme Court that respondent is not a real party in interest and dismissed the
case. Based on Rule 3 of the Rules of Court, a real party in interest is the party who stands to be
benefited or injured by the judgment of the suit. As applied in this case, respondent is not a party
to the contract and he also failed to show that he has a real interest which will be affected by its
performance or annulment. He also failed to prove that a trust was created between him and his
daughters. The legal presumption is that it was a gift effected in their favor, based on Article 1448
of the Civil Code.
18. ELPIDIO S. UY, doing business under the name and style EDISON
DEVELOPMENT & CONSTRUCTION
VS. COURT OF APPEALS AND THE
HERITAGE PARK MANAGEMENT CORPORATION (HPMC),
G.R. NO.157065,
QUISUMBING, J
FACTS:

JULY 11,2006

The Public Estates Authority (PEA) and the petitioner executed a Landscaping and

Construction Agreement whereby the petitioner undertook to do all the


landscaping, including the construction of a terrasoleum of the Heritage
Park. Alleging delay in the construction of the projects and huge discrepancy
between the Accomplishment Report and the actual physical accomplishment
of petitioners construction firm, the Heritage Park Executive Committee
terminated the construction contracts. Pursuant to the terms of the PFTA,
HPMC assumed all the functions, duties and responsibilities of the PEA,
including those under an assailed contract. Petitioner filed a complaint
against the PEA before the Construction Industry Arbitration Commission
(CIAC) where it sought to recover payment for its progress billings on the said
projects. The CIAC rendered decision in favor of the petitioner. Private respondent
HPMC then filed a petition for Injunction/Prohibition before the CA on the ground that CIAC had
no jurisdiction over the subject matter since HPMC was not impleaded as a party thereby
depriving it of its right to be heard. The appellate court ruled in favor of respondent.
Issue:

Whether or not HPMC is a real party-in-interest or an indispensable party?


Held:

YES. An indispensable party is one whose interest will be affected by the


courts action in the litigation, and without whom no final determination of the
case can be had. The partys interest in the subject matter of the suit and in
the relief sought are so inextricably intertwined with the other parties that his
legal presence as a party to the proceeding is an absolute necessity.
In the case at bar, PEA assigned its interests in all the existing contracts it
entered into as the Project Manager for Heritage Park to HPMC. PEA officially
turned over to HPMC all the documents and equipment in its possession
related to the Heritage Park Project. Petitioner was duly informed of these
incidents through a letter. Apparently, as of the date of the filing of the CIAC
Case, PEA is no longer a party-in-interest. Instead, it is now private
respondent HPMC, as the assignee, who stands to be benefited or injured by
the judgment in the suit. In its absence, there cannot be a resolution of the
dispute of the parties before the court which is effective, complete or
equitable. Hence, HPMC is an indispensable party.
19 Purok Bagong Silang vs Yuipco
GR No. 135092 May 4, 2006
Callejo, Sr., J.:

FACTS:
The Kaimos and their siblings are co-owners of a three parcels of land. However, about
400 persons constructed their house and other improvements in the property. The occupants
then formed an association known as Purok Bagong Silang Association, Inc. (PBSAI).

The Kaimos then filed a complaint for the recovery of possession of real property,
damages and attorneys fees against 64 occupants. The case was docketed as Civil Case No. 3203.
The occupants argued that the land in question is classified as timberland, and as such,
part of the public domain; thus, plaintiffs has no cause of action against them. Seven other
defendants declared that they stopped paying rents after knowing that the land in classified as
timberland. The trial court held in favor of the defendants, no appeal was made by the occupants;
thus, the decision became final and executory. However, the occupants refused to remove their
houses and improvements on the property and to vacate the same despite demands.
A trial court issued a writ of demolition and copies was served by the sheriff. However, the
sheriff served copies not only to the petitioners but also on 309 other individuals who are not a
party to Civil Case No. 3203. This prompted the eight individuals to file a petition for certiorari to
the CA which in turn issued a temporary restraining order enjoining the enforcement of the writ
of demolition. The sheriff tried to implement the writ of demolition in two occasions but failed
due to blockage made by the residents and through a temporary restraining order shown to him.
Hence, this petition claiming that respondent judge acted with grave abuse of discretion
amounting to lack or excess of jurisdiction. The petitioners further allege that not being a party to
the Civil Case No. 3203, it cannot appeal from the said questioned order and that it had no plain,
speedy and adequate remedy therefrom in the ordinary course of law except through the filing of
the petition.
ISSUE:
WON the petitioner is the real party-in-interest in this case.
WON the petition filed in the SC is appropriate

HELD:
I.

Yes, petitioner is the real party-in-interest in this case.

The rule is that all actions must be prosecuted and defended by the real parties-ininterest and in the name of the real party-in-interest. The party whose legal right has been
invaded or infringed or who sustained an injury is the only one who can maintain the action; or
the party who stands to be benefited or injured by the judgment in the suit. He must appear to be
the present owner of the right sought to be enforced.
An association has standing to file suit for its members despite its lack of direct interest if
its members are affected by the action; similarly, an organization has standing to assert the
concern of its constituents.
II. No, the petition filed in the SC is not appropriate.
Issuance of an extraordinary writ is also within the competence of the CA or the RTC, it is
in either of these courts that the specific action for the issuance of such writ must be proscribed
unless special and important laws are clearly and specifically set forth in the petition.
The case shall fall on the ground that petitioner immediately sought relief from this Court
for a writ of prohibition.

20. Arcadio and Maria Luisa Carandang vs. Heirs of Quirino de


Guzman

G.R. No. 160347

November 29, 2006

Chico-Nazario, J
[Quirino de Guzman] and [the Spouses Carandang] are stockholders as
well as corporate officers of Mabuhay Broadcasting System (MBS for
brevity), with equities at fifty four percent (54%) and forty six percent
(46%) respectively.
On November 26, 1983, the capital stock of MBS was increased, from
P500,000 to P1.5 million and P345,000 of this increase was subscribed
by [the spouses Carandang]. Thereafter, onMarch 3, 1989, MBS again
increased its capital stock, from P1.5 million to P3 million, [the spouses
Carandang] yet again subscribed to the increase. They subscribed to
P93,750 worth of newly issued capital stock.
[De Guzman] claims that, part of the payment for these subscriptions
were paid by him, P293,250 for the November 26, 1983 capital stock
increase and P43,125 for the March 3, 1989 Capital Stock increase or a
total of P336,375. Thus, on March 31, 1992, [de Guzman] sent a
demand letter to [the spouses Carandang] for the payment of said total
amount.
[The spouses Carandang] refused to pay the amount, contending that a
pre-incorporation agreement was executed between [Arcadio
Carandang] and [de Guzman], whereby the latter promised to pay for
the stock subscriptions of the former without cost, in consideration for
[Arcadio Carandangs] technical expertise, his newly purchased
equipment,
and
his
skill
in
repairing
and
upgrading
radio/communication equipment therefore, there is no indebtedness on
their part [sic].
De Guzman filed their complaint for the collection of the debts
allegedly incurred by the Carandangs. For their part, the Carandangs
claim that the case should be dismissed for an absence of a cause of
action since Milagros de Guzman, to whom which the checks were
issued by the Carandangs were named, was not included as a plaintiff
in the case since she is an indispensable party.
Issue:

Whether or Not the case can be dismissed due to the non-inclusion of


Milagros de Guzman.
Ruling:
No. Though the spouses Carandang were correct in invoking the
aforementioned doctrine, the ground set forth entails an examination of
whether the parties presently pleaded are interested in the outcome
of the litigation, and not whether all persons interested in such
outcome are actually pleaded. The first query seeks to answer the
question of whether Milagros is a real party in interest, while the latter
query is asking if she is an indispensable party. Since the issue of this
case calls for the definition of an indispensable party, invoking the
abovementioned doctrine is irrelevant to the case because the doctrine
talks about a real party in interest and not an indispensable party.
Although it is important to take note that an indispensable party is also
a real party in interest.
Moreover, under the Civil Code, since Milagros and Quirino de Guzman are
spouses thus, the credit in question is a conjugal property. Therefore, an
action instituted by the husband is sufficient.
21. ALLIANCE FOR RURAL AND AGRARIAN RECONSTRUCTION, INC v.
COMMISSION ON ELECTIONS
FACTS:
Petitioner, ARARO was a duly accredited party-list garnered a total of 147,204
votes in the May 10, 2010 elections and ranked 50th. The COMELEC En Banc
sitting as the National Board of Canvassers initially proclaimed twenty-eight
(28) party-list organizations as winners involving a total of thirty-five (35)
seats guaranteed and additional seats. The petitioner questioned the formula
used by the COMELEC and filed the present Petition for Review on Certiorari
with Prayer for Preliminary Injunction and Temporary Restraining Order
The petitioner suggests that the formula used by the Commission on
Elections is flawed because votes that were spoiled or that were not made for
any party-lists were not counted. According to the petitioner, around seven
million (7,000,000) votes were disregarded as a result of the Commission on
Elections
erroneous
interpretation. 7,112,792 (Total
number
of
disregarded votes according to petitioner ARARO)
On the other hand, the formula used by the Commission on Elections En
Banc sitting as the National Board of Canvassers is the following:
Number of seats available to legislative districts_x .20 =Number of seats
available to party-list representatives .80
Thus, the total number of party-list seats available for the May 2010 elections
is 57 as shown below:

229__x .20 =57 .80


The National Board of Canvassers Resolution No. 10-009 applies the formula
used in Barangay Association for National Advancement and Transparency
(BANAT) v. COMELEC18 to arrive at the winning party-list groups and their
guaranteed seats, where:
Number of votes of party-list
______________________________=
Proportion or Percentage of votes garnered by party-list
Total number of votes for party-list candidates
the Commission on Elections through the Office of the Solicitor General took
the position that invalid or stray votes should not be counted in determining
the divisor. The Commission on Elections argues that this will
contradict Citizens
Battle
Against
Corruption
(CIBAC)
v.
22
COMELEC and Barangay Association for National Advancement and
Transparency (BANAT) v. COMELEC.23 It asserts that:
Neither can the phrase be construed to include the number of voters who did
not even vote for any qualified party-list candidate, as these voters cannot be
considered to have cast any vote "for the party-list system.
Issue:

Whether petitioners have legal standing


Held:
"A real party in interest is the party who stands to be benefited or injured by
the judgement in the suit, or the party entitled to the avails of the suit." The
party's interest must be direct, substantial, and material.
However despite any new computation, ARAROs proposed divisor of total
votes cast for the party-list system whether valid or invalid still fails to secure
one seat for ARARO. Petitioner does not suffer a direct, substantial or material
injury from the application of the formula interpreted and used in BANAT in
proclaiming the winning party-lists in the assailed National Board of
Canvassers Resolution. The computation proposed by petitioner ARARO even
lowers its chances to meet the 2% threshold required by law for a guaranteed
seat. Its arguments will neither benefit nor injure the party. Thus, it has no
legal standing to raise the argument in this Court.
22. GENESIS INVESTMENT, INC. Vs HEIRS of CEFERINO EBARASABAL
G.R. No. 181622, November 20, 2013
FACTS:
Plaintiffs are surviving descendants either as grandchildren or great grandchildren
and heirs and successors-in-interest of deceased Roman Ebarsabal, who died on 07 September

1952. During the lifetime of Roman Ebarsabal, he acquired a parcel of land situated in Basdaku,
Saavedra, Moalboal, Cebu.
Upon the death of said Roman Ebarsabal, his eight (8) children namely Gil, Ceferino,
Floro, Leona, Pedro, Isidoro, Julian and Benito, all surnamed Ebarsabal. became co-owners of
property by hereditary succession; all of them likewise died, leaving, however, their respective
children and descendants and/or surviving heirs and successors-in-interest, and who are now the
plaintiffs
on 28th January 1997, the children and descendants of deceased Gil Ebarsabal, namely:
Pelagio, Hipolito, Precela, Fructuosa, Roberta, Florentino, Erlinda, Sebastian, Cirilo, all surnamed
Ebarsabal, have executed among themselves a Deed of Extrajudicial Settlement with Sale of
Roman Ebarsabal's entire property . They sold the whole property to defendants Genesis
Investment Inc. represented by co-defendant Rhodora B. Lim, the wife of Lambert Lim, without
the knowledge, permission and consent of the plaintiffs who are the vendors' co-owners of the lot
in question.
Respondents filed against herein petitioners a Complaint for Declaration of Nullity of
Documents, Recovery of Shares, Partition, Damages and Attorney's Fees. The Complaint was filed
with the Regional Trial Court (RTC) of Barili, Cebu.
On August 5, 2004, herein petitioners filed a Motion to Dismiss 4contending, among
others, that the RTC has no jurisdiction to try the case on the ground that, as the case involves
title to or possession of real property or any interest therein and since the assessed value of the
subject property does not exceed P20,000.00 (the same being only P11,990.00), the action falls
within the jurisdiction of the Municipal Trial Court (MTC).
Respondents filed a Motion for Partial Reconsideration, arguing that their complaint
consists of several causes of action, including one for annulment of documents, which is incapable
of pecuniary estimation and, as such, falls within the jurisdiction of the RTC.
RTC ruled in favor of the heirs of Ebarasabal. It said that main case or the primary relief
prayed for by the movants is for the declaration of nullity or annulment of documents which
unquestionably is incapable of pecuniary estimation and thus within the exclusive original
jurisdiction of this court.
The court of appeals affirmed the decision of the RTC.
Issue:
Whether or not this is a case of joinder of causes of action
Ruling:
Clearly, this is a case of joinder of causes of action which comprehends more than the
issue of partition of or recovery of shares or interest over the real property in question but
includes an action for declaration of nullity of contracts and documents which is incapable of
pecuniary estimation.
Contrary to petitioners contention, the principal relief sought by petitioners is the
nullification of the subject Extrajudicial Settlement with Sale entered into by and between some of
their co-heirs and respondents, insofar as their individual shares in the subject property are
concerned. Thus, the recovery of their undivided shares or interest over the disputed lot, which

were included in the sale, simply becomes a necessary consequence if the above deed is nullified.
Hence, since the principal action sought in respondents Complaint is something other than the
recovery of a sum of money, the action is incapable of pecuniary estimation and, thus, cognizable
by the RTC. Well entrenched is the rule that jurisdiction over the subject matter of a case is
conferred by law and is determined by the allegations in the complaint and the character of the
relief sought, irrespective of whether the party is entitled to all or some of the claims asserted.
It is provided under Section 5 (c), Rule 2 of the Rules of Court that where the causes of
action are between the same parties but pertain to different venues or jurisdictions, the joinder
may be allowed in the RTC provided one of the causes of action falls within the jurisdiction of said
court and the venue lies therein. Thus, as shown above, respondents complaint clearly falls within
the jurisdiction of the RTC.
Sc affirmed the decision of the CA.
23. Unicapital Inc., Unicapital Realty, Inc., and Jaime Martirez vs. Rafael Consing,
Jr. and the Presiding Judge of the Regional Trial Court of Pasig City, Branch 168
G.R. Nos. 175277 & 175285

September 11, 2013

Perlas-Bernabe, J.:
FACTS:
Rafael Consing, Jr., an investment broker, and his mother Cecilia obtained a loan of
P18,000,000.00 from Unicapital, Inc. (Unicapital). The loan was secured by promissory notes
and a real estate mortgage. Prior to these transactions, Plus Builders Inc. (PBI) entered into a
joint venture agreement with Unicapital through its real estate development arm, United Capital
Realty, Inc. (URI).
The loan was later on modified into an option to buy real property. Cecilia sold the property
to Unicapital and PBI. Before the titles were transferred to Unicapital and PBI, Juanito Teng and
Po Yu informed Unicapital that they were the lawful owners of the property and Cecilias title was
a mere forgery. Demand letters were sent to Cecilia and Consing Jr. for the return of the purchase
price of the property.
Consing Jr. filed a complaint before the Regional Trial Court (RTC) of Pasig against
Unicapital, URI and PBI. He claim that the demands made by Unicapital and PBI constituted
harassment and oppression which severely affected his personal and professional life. The latter
filed separate motions to dismiss. These were denied by the RTC. They then elevated the denial of
their motions to dismiss to the Court of Appeals (CA) which affirmed the RTCs decision.
ISSUE:
Whether or not there is a misjoinder of causes of action
HELD:
The causes of action were not misjoined even if in the complaint filed by Consing, Jr., he
averred that Unicapital and PBI et. al. violated certain provisions of the Corporation Law and the
Revised Securities Act.
The rule is that a partys failure to observe the conditions under Section 5, Rule 2 of the
Rules results in a misjoinder of causes of action. A careful perusal of his complaint discloses that
Consing, Jr. did not seek to hold Unicapital and PBI, et al. liable for any specific violation of the

Corporation Code or the Revised Securities Act. Rather, he merely sought damages for Unicapital
and PBI, et al.s alleged acts of making him sign numerous documents and their use of the same
against him. In this respect, Consing, Jr. actually advances an injunction and damages case which
properly falls under the jurisdiction of the RTC-Pasig City. Therefore, there was no violation of
Section 5, Rule 2 of the Rules, particularly, paragraph (c) thereof.

24. Boston Equity versus Court of Appeals


G.R. No. 173946

June 19, 2013

Perez, J.
Facts:
On 24 December 1997, petitioner filed a complaint for sum of money with a
prayer for the issuance of a writ of preliminary attachment against the
spouses Manuel and Lolita Toledo. Herein respondent filed an Answer dated
19 March 1998 but on 7 May 1998, she filed a Motion for Leave to Admit
Amended Answer in which she alleged, among others, that her husband and
co-defendant, Manuel Toledo (Manuel), is already dead.
Petitioner then filed a motion, to require respondent to disclose the heirs of
Manuel. In compliance with the verbal order of the court during the 11
October 1999 hearing of the case, respondent submitted the required names
and addresses of the heirs. Subsequently, Petitioner filed a Motion for
Substitution praying that Manuel be substituted by his children as partydefendants which was granted by the court.
On 26 May 2004, the reception of evidence for herein respondent was
cancelled upon agreement of the parties. On 24 September 2004, counsel for
herein respondent was given a period of fifteen days within which to file a
demurrer to evidence. However, on 7 October 2004, respondent instead filed
a motion to dismiss the complaint on the ground that: (1) that the complaint
failed to implead an indispensable party or a real party in interest; hence, the
case must be dismissed for failure to state a cause of action; (2) that the trial
court did not acquire jurisdiction over the person of Manuel pursuant to
Section 5, Rule 86 of the Revised Rules of Court; xxx. The motion was
dismissed by the trial court for being filed out of time.
Issue:
Whether or not the inclusion of Manuel as party defendant is a misjoinder of
party.
Ruling:

No, the inclusion of Manuel in the complaint cannot be considered a


misjoinder.
Based on the last sentence of Section 11, Rule 3, a misjoined party must have
the capacity to sue or be sued in the event that the claim by or against the
misjoined party is pursued in a separate case. In this case, therefore, the
inclusion of Manuel in the complaint cannot be considered a misjoinder, as in
fact, the action would have proceeded against him had he been alive at the
time the collection case was filed by petitioner. This being the case, the
remedy provided by Section 11 of Rule 3 does not obtain here. The name of
Manuel as party-defendant cannot simply be dropped from the case. Instead,
the case should be dismissed since there was no valid service of summons or
voluntary submission on the part of Manuel. However, said dismissal is
without prejudice to the filing of the action against the estate of Manuel.
25. G.R. No. 173946
June 19, 2013
BOSTON EQUITY RESOURCES, INC., Petitioner,
vs.
COURT OF APPEALS AND LOLITA G. TOLEDO
PEREZ, J.:
JOINDER OF CAUSES OF ACTION
Factual antecedents:

On 24 December 1997, petitioner filed a complaint for sum of money


with a prayer for the issuance of a writ of preliminary attachment
against the spouses Manuel and Lolita Toledo.
Respondent filed an Answer dated 19 March 1998 but on 7 May 1998,
she filed a Motion for Leave to Admit Amended Answer 7 in which she
alleged, among others, that her husband and co-defendant, Manuel
Toledo (Manuel), is already dead.

Petitioner filed a motion, dated 5 August 1999, to require respondent to


disclose the heirs of Manuel. In compliance with the verbal order of the
court during the 11 October 1999 hearing of the case, respondent
submitted the required names and addresses of the heirs.

Petitioner then filed a Motion for Substitution, dated 18 January 2000,


praying that Manuel be substituted by his children as party-defendants
which was granted by the trial court.

On 26 May 2004, the reception of evidence for herein respondent was


cancelled upon agreement of the parties. On 24 September 2004,
counsel for herein respondent was given a period of fifteen days within
which to file a demurrer to evidence.

However, on 7 October 2004, respondent instead filed a motion to


dismiss the complaint, citing the following as grounds: (1) that the
complaint failed to implead an indispensable party or a real party in
interest; hence, the case must be dismissed for failure to state a cause
of action; (2) that the trial court did not acquire jurisdiction over the
person of Manuel pursuant to Section 5, Rule 86 of the Revised Rules of
Court; (3) that the trial court erred in ordering the substitution of the
deceased Manuel by his heirs; and (4) that the court must also dismiss
the case against Lolita Toledo in accordance with Section 6, Rule 86 of
the Rules of Court

Issue: Whether or not the inclusion of Manuel as party defendant is a


misjoinder of party
Held:
Section 11 of Rule 3 of the Rules of Court states that "neither misjoinder nor
non-joinder of parties is ground for dismissal of an action. Parties may be
dropped or added by order of the court on motion of any party or on its own
initiative at any stage of the action and on such terms as are just. Any claim
against a misjoined party may be severed and proceeded with
separately."Based on the last sentence of the afore-quoted provision of law, a
misjoined party must have the capacity to sue or be sued in the event that
the claim by or against the misjoined party is pursued in a separate case. In
this case, therefore, the inclusion of Manuel in the complaint cannot be
considered a misjoinder, as in fact, the action would have proceeded against
him had he been alive at the time the collection case was filed by petitioner.
This being the case, the remedy provided by Section 11 of Rule 3 does not
obtain here. The name of Manuel as party-defendant cannot simply be
dropped from the case. The Honorable Court has not acquired jurisdiction
over the person of Manuel since there was indeed no valid service of
summons insofar as Manuel is concerned. He died before the summons,
together with a copy of the complaint and its annexes could be served upon
him. As a result, the case, as against Manuel, must be dismissed.
In addition, the dismissal of the case against Manuel is further warranted by
Section 1 of Rule 3 of the Rules of Court, which states that: only natural or
juridical persons, or entities authorized by law may be parties in a civil
action." Considering that capacity to be sued is a correlative of the capacity
to sue, to the same extent, a decedent does not have the capacity to be sued
and may not be named a party defendant in a court action. Where the
defendant is neither a natural nor a juridical person or an entity authorized by
law, the complaint may be dismissed on the ground that the pleading

asserting the claim states no cause of action or for failure to state a cause of
action pursuant to Section 1(g) of Rule 16 of the Rules of Court, because a
complaint cannot possibly state a cause of action against one who cannot be
a party to a civil action.
Since the proper course of action against the wrongful inclusion of Manuel as
party-defendant is the dismissal of the case as against him, thus did the trial
court err when it ordered the substitution of Manuel by his heirs. Substitution
is proper only where the party to be substituted died during the pendency of
the case
Since Manuel was already dead at the time of the filing of the complaint, the
court never acquired jurisdiction over his person and, in effect, there was no
party to be substituted.
26. ROLANDO D. LAYUG vs COMMISSION ON ELECTIONS
G.R. No. 192984
PERLAS-BERNABE, J.:
FACTS: Petitioner Rolando Layug, filed pro se a Petition to disqualify Buhay Party-List from
participating in the May 10, 2010 elections and Brother Mike from being its nominee. He argued
that Buhay Party-List is a mere extension of the El Shaddai, which is a religious sect and neither
does Brother Mike, qualify as one who belongs to the marginalized and underrepresented sector
as required of party-list nominees COMELEC Resolution No. 88076. Layug received a copy of the
Answer only at the hearing conducted on April 20, 2010 after his lawyer, Atty. Rustico Gagate,
manifested that his client has not received the same. COMELEC Second Division issued a
Resolution denying the petition for lack of substantial evidence. A copy thereof was sent to Layug
via registered mail at #70 Dr. Pilapil Street, Barangay San Miguel, Pasig City. However, the mail
was returned unserved. The COMELEC second division found Layug to be a phantom petitioner
by seeing to it that pleadings, orders, and judicial notices addressed to him because the address he
gave and maintains is fictitious. Layug was deemed to have received on June 23, 2010 a copy of
the Resolution dated June 15, 2010 and there being no motion for reconsideration filed within
reglementary period, said Resolution was declared final and executory.

ISSUE: Whether or not the petitioner was denied due process

HELD: The petitioner was not denied due process. A party may sue or defend an action pro se.
Under Section 3, Rule 7 of the Rules of Court, every pleading must be signed by the party or
counsel representing him, stating in either his address which should not be a post office box.
Layug filed pro se both the Petitions to Disqualify and his Position Paper before the COMELEC
Second Division. In the Petition to Disqualify, he stated his address as #70 Dr. Pilapil Street,
Barangay San Miguel, Pasig City. Neither did the Position Paper that was subsequently filed pro
se indicate any forwarding address. From the fact alone that the address which Layug furnished
the COMELEC was incorrect, his pretensions regarding the validity of the proceedings and
promulgation of the Resolution for being in violation of his constitutional right to due process are

doomed to fail. His refusal to rectify the error despite knowledge thereof impels the court to
conclude the Layug deliberately stated an inexistent address with the end in view of delaying the
proceedings upon the plea of lack of due process.

27. G.R. No. 173987

February 25, 2012

PADILLA MERCADO, ZULUETA MERCADO, BONIFACIA MERCADO, DAMIAN


MERCADO and EMMANUEL MERCADO BASCUG, Petitioners,
vs.
SPOUSES AGUEDO ESPINA and LOURDES ESPINA, Respondents.
FACTS:
Petitioners filed with the Regional Trial Court (RTC) of Maasin, Southern Leyte, a Complaint for
Recovery of Property and Declaration of Nullity of Deed of Sale, Certificate of Title and Damages.
The case was docketed as Civil Case No. R-3147.
Petitioners alleged in their Complaint that they are the heirs of the late spouses Santiago and
Sofronia Mercado, who were the owners of a 338 square meter parcel of land located at the
Poblacion of the then Municipality of Maasin (now a city), in the Province of Southern Leyte; that
after the death of Santiago and Sofronia, they inherited the disputed lot, possessing the same as
owners; sometime in 1996, herein respondents claimed ownership over the subject parcel of land,
alleging that they bought the same from one Josefa Mercado Espina (Josefa) who, allegedly
employed fraudulent machinations to obtain a title (Original Certificate of Title No. 35) over the
subject property in her name.
Respondents filed a Motion to Dismiss on grounds that the RTC has no jurisdiction over the case
due to the failure of the complainant to state the assessed value of the property, that petitioners'
cause of action is barred by prescription, laches and indefeasibility of title, and that the complaint
does not state sufficient cause of action against respondents who are buyers in good faith.
The RTC denied the Motion to Dismiss and the subsequent motion for reconsideration by
respondents. The latter filed a special civil action for certiorari with the CA which was likewise
denied.
Meanwhile, petitioners, by leave of court, filed an Amended Complaint to include the assessed
value of the subject property. Respondents filed a Motion to Dismiss Amended Complaint on
grounds of prescription, laches, indefeasibility of title and lack of cause of action.
The RTC denied respondents' Motion to Dismiss Amended Complaint and their subsequent
motion for reconsideration. Respondents filed a special civil action for certiorari with the CA,
which was granted, ruling that the Complaint in Civil Case No. R-3147 is DISMISSED. The CA
held that the Amended Complaint failed to state a cause of action. Hence, this petition for review
on certiorari under Rule 45 of the Rules of Court assailing the CA Decision.
ISSUES:
WON the CA erred in granting respondents Motion to Dismiss Amended Complaint.
HELD:

No. The Supreme Court held that the CA is correct in finding that the Amended Complaint indeed
failed to state a cause of action.
The Court explained that under Section 8, Rule 10 of the Rules of Court, an amended complaint
supersedes an original one. As a consequence, the original complaint is deemed withdrawn and
no longer considered part of the record. The Amended Complaint is, thus, treated as an entirely
new complaint. As such, respondents had every right to move for the dismissal of the said
Amended Complaint. And so, failure to state a cause of action is a proper ground of the dismissal
of the Amended Complaint.
Failure to state a cause of action refers to the insufficiency of the pleading, and is a ground for
dismissal under Rule 16 of the Rules of Court.
A complaint states a cause of action if it avers the existence of the three essential elements of a
cause of action, namely:
(a) The legal right of the plaintiff;
(b) The correlative obligation of the defendant; and
(c) The act or omission of the defendant in violation of said legal right.
If the allegations in the complaint do not aver the concurrence of these elements, the complaint
becomes vulnerable to a motion to dismiss on the ground of failure to state a cause of action. A
perusal of the Amended Complaint in the present case would show that there is, indeed, no
allegation of any act or omission on the part of respondents which supposedly violated the legal
rights of petitioners. Thus, the CA is correct in dismissing the complaint on the ground of failure
to state a cause of action.
Hence, petition was denied.

28. Tapay vs. Bancolo (694 SCRA 1)


FACTS:
Rodrigo Tapay and Anthony Rustia, both employees of the Sugar Regulatory
Administration received an Order from the Office of the Ombudsman-Visayas requiring
them to file a counter-affidavit to a complaint for usurpation of authority, falsification of
public document, and graft and corrupt practices filed against them by Nehimias
Divinagracia, Jr., a co-employee. The Complaint was allegedly signed on behalf of
Divinagracia by Atty. Charlie L. Bancolo. When Atty. Bancolo and Rustia accidentally
chanced upon each other, the latter informed Atty. Bancolo of the case filed against
them. Atty. Bancolo denied that he represented Divinagracia since he had yet to meet
Divinagracia and declared that the signature in the Complaint was not his. Thus, Atty.
Bancolo signed an affidavit denying the said signature. This affidavit was used by Tapay and Rustia in
filing a counter-affidavit accusing Divinagracia of falsifying the signature of Atty. Bancolo. Divinagracia,
denying the same, presented as evidence an affidavit by Richard A. Cordero, the legal assistant of Atty.
Bancolo, that the Jarder Bancolo Law Office accepted Divinagracias case and that the Complaint filed
with the Office of the Ombudsman was signed by the office secretary per Atty. Bancolos instructions. The
case was then dismissed.
Tapay and Rustia then later filed with the Integrated Bar of the Philippines a complaint
to disbar Atty. Bancolo and Atty. Jarder, Atty. Bancolos law partner. The complainants

alleged that not only were respondents engaging in unprofessional and unethical
practices, they were also involved in falsification of documents used to harass and
persecute innocent people. In their Answer, respondents admitted that due to some
minor lapses, Atty. Bancolo permitted that the pleadings be signed in his name by the
secretary of the law office. After investigation, Atty. Lolita A. Quisumbing, the
Investigating Commissioner of the Commission on Bar Discipline of the IBP, submitted
her Report. Atty. Quisumbing found that Atty. Bancolo violated Rule 9.01 of Canon 9 of
the Code of Professional Responsibility while Atty. Jarder violated Rule 1.01 of Canon 1
of the same Code, and recommended that Atty. Bancolo be suspended for two years
from the practice of law and Atty. Jarder be admonished for his failure to exercise
certain responsibilities in their law firm
Issue: Whether or not a pleading signed by a non-lawyer is considered valid
Ruling: the Supreme Court emphasized that that the preparation and signing of a pleading constitutes legal
work involving the practice of law which is reserved exclusively for members of the legal profession.
While the signing of a pleading may be delegated to other lawyers, it cannot be delegated to non-lawyers
because under the Rule 7, Section 3 of the Rules of Court, counsels signature serves as a certification that
(1) he has read the pleading; (2) to the best of his knowledge, information and belief there is good ground
to support it; and (3) it is not interposed for delay. By affixing ones signature to a pleading, it is counsel
alone who has the responsibility to certify to these matters and give legal effect to the document.
29. LANDBANK VS CASTRO
Gr No. 189125 August 28, 2013
PONENTE: Justice Perez
FACTS:
Bienvenido Castro is the owner of an unregistered property in located in Surigao Del Sur. He
voluntarily offered to sell the property for the total price of Php 560,340.00 to the Department of
Agrarian Reform (DAR) under RA 6657 or the Comprehensive Agrarian Reform Law. DAR,
Landbank conducted an ocular inspection which assessed the value of the property for the total
amount of Php 144, 205. 90 which Castro rejected the offer. DARAB then conducted a summary
proceeding to fix the just compensation of the property which reached the same valuation. 2 years
later, Castro filed a petition to fix the assessed value before the RTC, sitting as SAC which was
granted. The court set the case for pre-trial and the parties submitted their pre-trial briefs.
Landbank filed a Motion for Reconsideration alleging that Castro had already accepted the assed
value as shown in the documents that Castro had signed which was denied. Trial court ruled that
the defense or objection is not one of recognized exceptions to the rule on waiver of defenses not
pleaded in the answer. Landbank then filed an appeal with CA which was denied. CA ruled that
the failure of Landbank to raise the defense is a procedural infirmity which cannot be cured on
appeal.
ISSUE: Whether or not the valuation of the courts of the property of Castro which he offered
under Contract to Sell is binding upon the parties.
RULING: Yes.
Since the statement in the pleading is conclusive to the pleader, it is unaffected by any
contrary proof submitted by the pleader, whether or not objection is interposed by any party. In

the case of Santiago vs Delos Santos, even if there had been a full hearing on the case therefore,
the result would not have been any different. There was no choice then for the lower court except
to dismiss the complaint.
In the case at bar, the payment by Landbank for the property and its transfer to the Republic
was fully discussed and submitted before the trial court through LBPs motion for
reconsideration. The trial court and appellate court incorrectly viewed the motion as procedurally
unacceptable. Rather it should be, a reminder that it is already conclusive on Castro as a pleader,
of transfer of ownership to the Republic.
30. CONSOLIDATED INDUSTRIAL GASES, INC., Petitioner, v. ALABANG MEDICAL
CENTER, Respondent.
G.R. No. 181983
November 13, 2013
REYES, J
FACTS:
Petitioner Consolidated Industrial Gases, Inc. (CIGI) is a domestic corporation engaged
in the business of selling industrial gases and installing centralized medical and vacuum pipeline
system. Respondent Alabang Medical Center, on the other hand, is a domestic corporation
operating a hospital business. CIGI, as contractor and AMC, as owner, entered into a contract
whereby the former bound itself to provide labor and materials for the installation of a medical
gas pipeline system for the first, second and third floors of the hospital for the contract price of
P9,856,725.18) which AMC duly paid in full. The legal controversy arose after the parties entered
into another agreement on October 3, 1996 for the continuation of the centralized medical oxygen
and vacuum pipeline system in the hospitals fourth & fifth floors at the cost P2,267,344.42 . This
second contract followed the same terms and conditions of the contract for the Phase 1
installation project. CIGI forthwith commenced installation works for Phase 2 while AMC paid
the partial amount of P1,000,000.00 with the agreement that the balance shall be paid through
progress billing and within fifteen days from the date of receipt of the original invoice sent by
CIGI.
CIGI sent AMC Charge Sales Invoice No. 125847 as completion billing for the unpaid
balance of P1,267,344.42 for the Phase 2 installation project. When the sales invoice was left
unheeded, CIGI sent a demand letter to AMC. AMC, however, still failed to pay thus prompting
CIGI to file a collection suit before the RTC. CIGI claimed that AMCs obligation to pay the
outstanding balance of the contract price for the Phase 2 installation project is already due and
demandable. In its Answer with Counterclaim,7 AMC averred that its obligation to pay the
balance of the contract price has not yet accrued because CIGI still has not turned over a complete
and functional medical oxygen and vacuum pipeline system. AMC filed a Motion for Leave of
Court to Admit Amended Answer with Counterclaims seeking, in addition, the rescission of the
subject contracts, return of its payment of P10,856,000.00 for an unfinished project. AMC also
asked that it be recompensed in the sum of P17,220,084.90 for interest expense on the loans
obtained from Metrobank which were used to fund the installation projects.
The RTC rendered its Decision, wherein it adjudged AMC to have breached the contract
for failure to perform its obligation of paying the remaining balance of the contract price. CIGI, on
the other hand, was found to have faithfully complied with its contractual obligations. The CA
ruled that it was CIGI who breached the contract when it failed to complete the project and to
turn over a fully functional centralized medical oxygen and vacuum pipeline system. The CA re-

examined its earlier decision and issued an Amended Decision dated March 4, 2008. It took into
consideration AMCs manifestation that it is willing to pay the balance of P1,267,344.42 on the
condition that CIGI will turn over a fully functional centralized medical oxygen and vacuum
pipeline system. The CA found that CIGI reneged on its obligation under the contract when it
failed to test run the installed system.
ISSUE: Whether or not AMC has legal basis to demand the rescission of the installation
contracts.
HELD:
AMC has no legal basis to demand the rescission of the installation contracts. Rescission was
pleaded in AMCs original Answer with Counterclaim when it implored the RTC for "other reliefs
and remedies consistent with law and equity are prayed for." The standing rule is that "the prayer
in the complaint for other reliefs equitable and just in the premises justifies the grant of a relief
not otherwise specifically prayed for." This rule conveys the inference that reliefs not specifically
pleaded but included in a general prayer for other equitable reliefs may be threshed out by the
courts. However, Rescission of a contract will not be permitted for a slight or casual breach, but
only for such substantial and fundamental violations as would defeat the very object of the parties
in making the agreement. Whether a breach is substantial is largely determined by the attendant
circumstances. The provisions on the test run of and seminar on the medical oxygen system are
not essential parts of the installation contracts as they do not constitute a vital fragment/part of
the centralized medical oxygen system. Further, the allegedly defective and incomplete parts
cannot substantiate rescission. The photographs submitted by AMC are not adequate to establish
that certain parts of the installed system are indeed defective or incomplete especially so that the
installation never became operational. Unless and until the medical oxygen and vacuum pipeline
actually runs, there is no way of conclusively verifying that some of its parts are defective or
incomplete. In addition, AMC failed to allege much less show whether the alleged defects and
incomplete components were caused by factory defect, negligence on the part of CIGI or ordinary
wear and tear.
#31 Swedish Match Phil Inc. v Treasurer of the City of Manila
G.R. No. 181277

July 3, 2013

Sereno, CJ.:

Facts:
On October 20, 2001, the petitioner paid business taxes amounting to P470,932.21 under the
Sections 14 and 21 of the Manila Revenue Code as amended. Out of the said total amount,
P164,552.04 corresponded to the payment under Section 21 of the said Code. Later, however, the
petitioner asserted that it was not liable to pay taxes under Section 21 and thus wrote a letter to
the respondent claiming a refund of business taxes that it had paid under the said section. It
argued that its payment under Section 21 constituted double taxation in view of its payment under
Section 14.
On October 17, 2003, for the alleged failure of the respondent to act on its claim for refund,
the petitioner filed a Petition for Refund of Taxes with the RTC of Manila. The RTC dismissed the

said petition for the failure of the petitioner to plead the latters capacity to sue and to state the
authority of Tiarra Beleno who had executed the Verification and Certification of Non-Forum
Shopping. On appeal, the CTA affirmed the RTCs dismissal on the same ground. The CTA En
Bank likewise denied the Petition for Review, stating that it was not shown that Ms. Beleno, the
companys Finance Manager, was given authority to act in behalf of the corporation at the time
the initiatory pleading was filed in the RTC.
The petitioner argued, however, that Ms. Beleno was acting within her authority in
instituting the Petition before the RTC notwithstanding that it was not accompanied by a
Secretarys Certificate. Her authority was ratified by the Board in its Resolution adopted on May
19, 2004. Thus, even if she was not authorized to execute the Verification and Certification at the
time of the filing of the Petition, the ratification by the Board of Directors retroactively applied to
the date of her signing.

Issue:
WON the case should be dismissed for the lack of authority of Ms. Beleno in instituting the
initiatory pleading in the RTC.

Held:
NO. It is true that a verification signed without authority from the board of directors is
defective. However, the requirement of verification is simply a condition affecting the form of the
pleading and non-compliance does not necessarily render the pleading fatally defective. The court
may in fact order the correction of the pleading if verification is lacking or it may act on the
pleading although it may not have been verified, where it is evident that strict compliance with the
rules may be dispensed with so that the ends of justice may be served.
32. Pasos vs. Philippine National Construction Corporation (Villarama, JR.,J.)
G.R Nos: 192394
Date: 7/3/13

Facts: Pasos was employed to the respondent and was on an extended employment. After the
termination of his contract, he was asked to report to work the next day followed by a medical
examination.
His medical exams diagnosed him with a disease and was asked to be on leave. Petitioner still
served a 60-day sick leave and underwent another medical examination on February 16, 2001. He
was then given a clean bill of health and was given a medical clearance by Dr. Obena that he was
fit to work. He was told that he was already terminated and replaced.
Respondent petitioner was hired as a project employee in several projects with specific dates of
engagement and termination and had full knowledge and consent that his appointment was only
for the duration of each project. It further contended that it had sufficiently complied with the
reportorial requirements to the Department of Labor and Employment (DOLE).

The Labor Arbiter decided that petitioner had obtained regular employment after repeated hiring
and rehiring of his servies.
Issue: whether the corporate officer may validly issue a certification.
Held: Yes. Even without a board resolution, a corporate officer may issue a verification and
certification on non-form shopping. Also, it was held by the court that in labor cases, procedural
rule must be relaxed.
It has been held that the following officials or employees of the company can sign the verification
and certification without need of a board resolution: (1) the Chairperson of the Board of Directors,
(2) the President of a corporation, (3) the General Manager or Acting General Manager, (4)
Personnel Officer, and (5) an Employment Specialist in a labor case.
While the above cases do not provide a complete listing of authorized signatories to the
verification and certification required by the rules, the determination of the sufficiency of the
authority was done on a case to case basis. The rationale applied in the foregoing cases is to justify
the authority of corporate officers or representatives of the corporation to sign the verification or
certificate against forum shopping, being "in a position to verify the truthfulness and correctness
of the allegations in the petition.

33. Pigcaualan v. Security and Credit Investigation, Inc.


G.R. No. 173648 , January 16, 2012
Del Castillo, J.
Facts:
Canoy and Pigcaulan were both employed by SCII as security guards and
were assigned to SCIIs different clients. Subsequently, however, Canoy and
Pigcaulan filed with the Labor Arbiter separate complaints 7 for underpayment
of salaries and non-payment of overtime, holiday, rest day, service incentive
leave and 13th month pays. These complaints were later on consolidated as
they involved the same causes of action.
Upon favorable decision in their cause of action before the Labor Arbiter and
the NLRC, CA Reversed the ruling of the NLRC which prompted Pigcaualan to
file a petition for certiorari before the SC.
It was noted that in the Present Case, it was only Pigcaualan who filed the
case since he was the sole petitioner who signed in the Certificate of NonForum Shopping.
Issue:
W/N the CA Decision is already binding on Canoy.
Ruling:

Yes, the CA Decision is binding on Canoy.


It appears from the heading of Parties that only Pigcaulan is mentioned as
petitioner and consistent with this, the body of the petition refers only to a
"petitioner" and never in its plural form "petitioners". Aside from the fact that
the Verification and Certification of Non-Forum Shopping attached to the
petition was executed by Pigcaulan alone, it was plainly and particularly
indicated under the name of the lawyer who prepared the same, Atty. Josefel
P. Grageda, that he is the "Counsel for Petitioner Adbuljuahid Pigcaulan" only.
In view of these, there is therefore, no doubt, that the petition was brought
only on behalf of Pigcaulan. Since no appeal from the CA Decision was
brought by Canoy, same has already become final and executory as to him.
34. G.R. No. 174082
January 16, 2012
GEORGIA T. ESTEL, Petitioner,
vs.
RECAREDO P. DIEGO, SR. and RECAREDO R. DIEGO, JR., Respondents.
PERALTA, J.
CERTIFICATION AGAINST FORUM SHOPPING
Factual antecedents:

The present petition originated from a Complaint for Forcible Entry,


Damages and Injunction with Application for Temporary Restraining
Order filed by herein respondents with the Municipal Trial Court in
Cities (MTCC) of Misamis Oriental.

Respondents alleged that on April 16, 1991, they entered into a


contract of sale of a parcel of land with petitioner; after receiving the
amount

of P17,000.00

as

downpayment,

petitioner

voluntarily

delivered the physical and material possession of the subject property


to respondents;

Respondents had been in actual, adverse and uninterrupted possession


of the subject lot since then and that petitioner never disturbed,
molested, annoyed nor vexed respondents with respect to their
possession of the said property;

Around 8:30 in the morning of July 20, 1995, petitioner, together with
her two grown-up sons and five other persons, uprooted the fence
surrounding the disputed lot, after which they entered its premises and
then cut and destroyed the trees and plants found therein; respondent
Recaredo R. Diego, Jr. witnessed the incident but found himself helpless
at that time.

On

July

26,

1995,

the

MTCC

issued

Temporary

Restraining

Order against petitioner and any person acting in her behalf.

On February 16, 2002, the MTCC rendered a Decision in favor of the


plaintiffs.

Aggrieved, petitioner appealed to the RTC of Gingoog City.

On October 7, 2002, the RTC rendered its Decision affirming the


assailed Decision of the MTCC.

Petitioner then filed a petition for review with the CA.

On September 30, 2005, the CA promulgated its Decision which


affirmed the Decision of the RTC.

Petitioner filed a Motion for Reconsideration, but the CA denied it in its


Resolution dated August 10, 2006.

Petitioner also avers that the complaint states no cause of action


because the verification and certificate of non-forum shopping
accompanying the complaint are defective and, as such, the complaint
should be treated as an unsigned pleading.

With respect to the certificate of non-forum shopping, petitioner claims


that its defect consists in respondents' failure to make an undertaking
therein that if they should learn that a similar action or proceeding has
been filed or is pending before the Supreme Court, the Court of
Appeals or any other tribunal or agency, they shall report that fact
within five (5) days therefrom to the court or agency wherein the
original pleading and sworn certification have been filed.

Issue:
Is there a defect with respect to the certificate of non-forum shopping that
shall order the dismissal of the case?
Ruling:

None. As to respondents' certification on non-forum shopping, a reading of


respondents Verification/Certification reveals that they, in fact, certified
therein that they have not commenced any similar action before any other
court or tribunal and to the best of their knowledge no such other action is
pending therein. The only missing statement is respondents' undertaking that
if they should thereafter learn that the same or similar action has been filed
or is pending, they shall report such fact to the court. This, notwithstanding,
the Court finds that there has been substantial compliance on the part of
respondents.
It is settled that with respect to the contents of the certification against forum
shopping, the rule of substantial compliance may be availed of. This is
because the requirement of strict compliance with the provisions regarding
the certification of non-forum shopping merely underscores its mandatory
nature in that the certification cannot be altogether dispensed with or its
requirements completely disregarded. It does not thereby interdict substantial
compliance with its provisions under justifiable circumstances, as the Court
finds in the instant case.

35. G.R. No. 179488


CASE: COSCO SHIPPING PHILS, INC. V KEMPER INSURANCE CO (670 SCRA
343)
CASE DIGEST:
FACTS

Kemper Insurance Company is a foreign insurance company based in Illinois,


United States of America (USA) with no license to engage in business in the
Philippines, as it is not doing business in the Philippines, except in isolated
transactions; while petitioner is a domestic shipping company organized in
accordance with Philippine laws.
In 1998 Kemper insured the shipment of imported frozen boneless beef
(owned by Genosi, Inc. Upon upon arrival at the Manila port, a portion of the
shipment was rejected by Genosi, Inc. by reason of spoilage arising from the

alleged temperature fluctuations of COSCO Shipping Phils. reefer containers.


Genosi, Inc. then filed a claim against both Cosco and Kemper. Thereafter,
Kemper paid the claim of Genosi. Hence, in 1999. Kemper filed a complaint
for insurance loss and damage against Cosco alleging that that despite the
repeated demands, Cosco failed and refused to pay the value loss sustained
due to the fault of Cosco's container.
In response, Cosco insisted that Kemper had no capacity to sue since it was
doing business in the Philippines without the required license. Petitioner filed
a motion to dismiss contending that the same was filed by Atty. Lat who failed
to show his authority to sue and sign the corresponding certification against
forum shopping.
2002, the trial court granted petitioner's Motion to Dismiss and dismissed the
case without prejudice, ruling that it is mandatory that the certification must
be executed by the petitioner himself, and not by counsel. Since respondent's
counsel did not have a Special Power of Attorney (SPA) to act on its behalf,
hence, the certification against forum shopping executed by said counsel was
fatally defective and constituted a valid cause for dismissal of the complaint.
On appeal by respondent, the CA ruled that the required certificate of nonforum shopping is mandatory and that the same must be signed by the
plaintiff or principal party concerned and not by counsel; and in case of
corporations, the physical act of signing may be performed in behalf of the
corporate entity by specifically authorized individuals. However, the CA
pointed out that the factual circumstances of the case warranted the liberal
application of the rules and, as such, ordered the remand of the case to the
trial court for further proceedings. Petitioners Motion for Reconsideration was
later denied by the CA.
ISSUE
Whether or not Atty. Rodolfo Lat was properly authorized by respondent to sign the certificate of
non-forum shopping on its behalf.

RULING
Petition is meritorious

Contrary to the CA's finding, the Court finds that the circumstances of this
case do not necessitate the relaxation of the rules. There was no proof of
authority submitted, even belatedly, to show subsequent compliance with the
requirement of the law. Neither was there a copy of the board resolution or
secretary's certificate subsequently submitted to the trial court that would
attest to the fact that Atty. Lat was indeed authorized to file said complaint
and sign the verification and certification against forum shopping, nor did
respondent satisfactorily explain why it failed to comply with the rules. Thus,
there exists no cogent reason for the relaxation of the rule on this
matter. Obedience to the requirements of procedural rules is needed if we are
to expect fair results therefrom, and utter disregard of the rules cannot justly
be rationalized by harking on the policy of liberal construction.
We have consistently held that the certification against forum shopping must
be signed by the principal parties. If, for any reason, the principal party
cannot sign the petition, the one signing on his behalf must have been duly
authorized. With respect to a corporation, the certification against forum
shopping may be signed for and on its behalf, by a specifically authorized
lawyer who has personal knowledge of the facts required to be disclosed in
such document. A corporation has no power, except those expressly
conferred on it by the Corporation Code and those that are implied or
incidental to its existence. In turn, a corporation exercises said powers
through its board of directors and/or its duly authorized officers and agents.
Thus, it has been observed that the power of a corporation to sue and be
sued in any court is lodged with the board of directors that exercises its
corporate powers. In turn, physical acts of the corporation, like the signing of
documents, can be performed only by natural persons duly authorized for the
purpose by corporate by-laws or by a specific act of the board of directors.

In the present case, since respondent is a corporation, the certification must


be executed by an officer or member of the board of directors or by one who
is duly authorized by a resolution of the board of directors; otherwise, the
complaint will have to be dismissed. The lack of certification against forum
shopping is generally not curable by mere amendment of the complaint, but
shall be a cause for the dismissal of the case without prejudice. The same

rule applies to certifications against forum shopping signed by a person on


behalf of a corporation which are unaccompanied by proof that said signatory
is authorized to file the complaint on behalf of the corporation.
The decision of CA are reversed and set aside while the orders of the RTC are reinstated.

36. Medado vs. Heirs of Antonio Consing (665 SCRA 534) Camacho
37. Otero vs. Tan (678 SCRA 583)
DEFAULT

FACTS:
A Complaint for collection of sum of money and damages was filed by Roger Tan (Tan)
with the Municipal Trial Court in Cities (MTCC)against Roberto Otero (Otero) alleging that on
several occasions from February 2000 to May 2001, Otero purchased on credit petroleum
products from his Petron outlet in Valencia City, Bukidnon in the aggregate amount of
P270,818.01. He further claimed that despite several verbal demands, Otero failed to settle his
obligation. Despite receipt of the summons and a copy of the said complaint, which
per the records of the case below were served through his wife Grace R. Otero on
August 31, 2005, Otero failed to file his answer with the MTCC.
On November 18, 2005, Tan filed a motion with the MTCC to declare Otero in
default for his failure to file his answer. Otero opposed Tans motion, claiming that
he did not receive a copy of the summons and a copy of Tans complaint. Hearing on
the said motion was set on January 25, 2006, but was later reset to March 8, 2006, Otero
manifesting that he only received the notice therefor on January 23, 2006.
The hearing on March 8, 2006 was further reset to April 26, 2006 since the presiding
judge was attending a convention. Otero failed to appear at the next scheduled hearing,
and the MTCC issued an order declaring him in default. A copy of the said order was sent
to Otero on May 9, 2006. Tan was then allowed to present his evidence ex parte.
On February 14, 2007, the MTCC rendered a Decision directing Otero to pay Tan his
outstanding obligation and opined that Oteros failure to file an answer despite notice is a tacit
admission of Tans claim. Otero appealed the MTCC Decision dated February 14, 2007 to the RTC
but the latter affirmed the decision of the lower court. Hence, Otero appealed to the CA.

ISSUE: W/N the order declaring Otero in default is correct.


Ruling: YES.
CA held that any defense which Otero may have against Tans claim is already deemed
waived due to Oteros failure to file his answer. Thus: Otero never denied that his wife received
the summons and a copy of the complaint. He did not question the validity of the substituted

service. Consequently, he is charged with the knowledge of Tans monetary claim. Section 1, Rule
9 of the Rules of Court explicitly provides that defenses and objections not pleaded are deemed
waived. Moreover, when the defendant is declared in default, the court shall proceed to render
judgment granting the claimant such relief as his pleading may warrant. Due to Oteros failure to
file his Answer despite being duly served with summons coupled with his voluntary appearance in
court, he is deemed to have waived whatever defenses he has against Tans claim. Apparently,
Otero is employing dilatory moves to defer the payment of his obligation which he never denied.

38. G.R. No. L-63397


LINA v. COURT OF APPEALS (135 SCRA 637)
CASE DIGEST:
FACTS:
On 31 March 1982, private respondent Northern Motors, Inc. filed with CFI of Rizal (Pasig) a case
for sum of money which was served with summons on April 22, 1982. When no answer or motion
to dismiss was filed by petitioner, private respondent filed a motion to declare him in default
which was set for hearing on May 21, 1982. On May 19, 1982, petitioner filed his opposition to the
aforesaid motion inviting attention to the fact that he had filed a motion for extension of time to
file a responsive pleading within the reglementary period. However, on 26 May, 1982, respondent
judge issued an order declaring defendant in default and allowing plaintiff to adduce its evidence
ex parte. May 27, 1982, petitioner filed his answer however the court rendered its decision in
favour of plaintiff. Later petitioner filed a motion to set aside the decision which was denied by
respondent judge. Hence, petitioner filed with the CA for certiorari/prohibition.
ISSUE:
Whether or not the order of default was issued in grave abuse of discretion amounting to lack of
jurisdiction.
HELD:
NO. The granting of additional time within which to file an answer to a complaint is a matter
largely addressed to the sound discretion of the trial court. In NDC vs CA, the court stated that
while trial courts are persuaded, as a matter of policy, to adopt a basically flexible attitude in
favour of the defendant in this area of our adjective law, the defense should never be lulled the
belief that whenever trial courts refuse a second request for extension to file an answer, the
appellate courts will grant relief.
In the case at bar, it was on May 5, 1982 or two (2) days before the expiration of the fifteen-day
reglementary period given to defendant to file his responsive pleading when petitioner moved for
an extension of twenty (20) days from May 7 within which to file his answer. Upon motion of
private respondent and over the objection of petitioner, respondent judge issued an order
declaring petitioner in default.
Petitioner in this case did not avail himself of any of the remedies. Instead, he went to the
appellate court on certiorari/prohibition. On this point, respondent appellate court aptly said ...
where the judgment rendered by the respondent court is the one sought to be annulled, a petition

for relief, under Rule 38 of the Revised Rules of Court, which is a remedy in the ordinary course of
law, could have been just as plain, adequate and speedy as certiorari. Such a remedy could have
been granted by the respondent court. And if the respondent court still denies the petition, then
petitioner can take an appeal on the order denying the petition, and in the course of such appeal
petitioner can also assail the judgment an the merits upon the ground that it is supported by the
evidence, or it is contrary to law.
#39 Magtoto v. CA
G.R. No. 175792, November 21, 2012
DEL CASTILLO, J.
Facts:
A contract of sale of 3 parcels of land were entered into between spouses Magtoto and Leonila.
The spouses made payment in the form of checks. The TCTs were delivered to the spouses.
However, most of the checks given by the spouses were dishonored. Thus, Leonila demanded for
the balance of the purchase price from the spouses but to no avail. Hence, Leonila filed a
complaint for specific performance with damages against the spouses before the RTC. Service of
summons was made. The spouses, however, thrice moved for extensions of time to file an Answer,
until a final extention was given to them by the RTC. Two days after the last day for filing an
Answer, the spouses instead filed a Motion to dismiss which the RTC denied for lack of merit.
Later, Atty Canlas, the counsel for the spouses filed an Ex-Parte Motion to Withdraw Appearance
as counsel for petitioners. The motion was set for hearing but Atty. Canlas did not appear.
Subsequently, the spouses were decalred in default, upon motion filed by Leonila. During the
hearing of said motion, the court a quo noted that despite the spouses counsels withdrawal, the
spouses have not yet engaged the services of a new counsel. Alost three months after they were
declared in default, the spouses, through their new counsel, filed an Omnibus Motion to Lift
Order of Default. RTC denied said motion holding that there is no showing of fraud, accident,
mistake, or inexcusable negligence negligence to grant the motion. Thus, a decision was rendered
by the RTC, ruling in favor of Leonila. Upon appeal, CA dismissed the same for being bereft of
merit. MR denied.
Issue:
WON the CA correctly declared efendants in default.
Held: Yes. The spouses Magtotos failure to file atimely Answer was due to their ownfault.
petitioners should have filed their Answer within the balance of the period prescribed in Rule 11.
Instead, they filed their Answer on June 25, 2004 or nine months after the denial of their Motion
to Dismiss or three months after they were declared in default. This delay is unreasonable as well
as unjustified. They miserably failed to be vigilant in protecting and defending their
cause.1wphi1 The RTC thus properly declared them in default. Furthermore, the spouses
Magtoto are unable to show that their failure to timely file an Answer was due to fraud, accident,
mistake or excusable negligence and, more importantly, that they have a meritorious defense
pursuant to Section 3(b), Rule 9 of the Rules of Court. "Negligence, to be excusable, must be one
which ordinary diligence and prudence could not have granted against." Certainly, this is not the
kind of negligence committed by the spouses Magtoto in this case. More significantly, a review of
the records does not convince the Court that the spouses Magtoto have a meritorious defense. At
most, the allegations in their Answer and the attached Affidavit of Merit, to wit that the agreed
purchase price is only P10,000,000.00; that they provided financial support to Leonila for the
settlement of estate of the latters predecessors-in-interest and for the transfer of titles in her
name; and that they already paid the total amount of P4,500,000.00, are mere allegations not
supported by evidence, at the outset, are supposed to present.

40. Magdiwang Realty vs The Manila Banking Corporation


G.R. No. 195592 September 05, 2012
SERENO, CJ
FACTS:
TMBC filed a complaint for a sum of money against Magdiwang, Dragon, and Tolentino in the
RTC. TMBC alleged that the petitioners failed to attend to the banks several demands for
payment. Instead of filing a responsive pleading with the trial court, the petitioners filed beyond
the fifteen days period allowed for the filing of a responsive pleading, a Motion for leave to Admit
Attached Motion to Dismiss and a Motion to Dismiss. The motions were opposed by the
respondent TMBC and contended that the petitioners be declared in default for their failure to file
their responsive pleading within the period allowed under the law. The RTC declared the
petitioners in default in which the CA affirmed.
ISSUE:
Whether or not the petitioners failure to file any responsive pleading within the 15 days provided
under the Rules of Court placed them in default.
HELD:
Yes. The petitioners, in accordance with the rules, should have filed an Answer or a Motion to
Dismiss or any responsive pleading within the prescribed period, which is fifteen days from
receipt of Summons and a copy of the complaint with attached annexes. In this case, the
petitioners were not able to file their responsive pleading hence, giving sufficient basis to declare
the petitioners in default as provided by Section 3 of Rule 9 of the Revised Rules of Court.
41. Ada vs. Baylon (678 SCRA 293)
G.R. No. 182435
This case involves the estate of spouses Florentino Baylon and Maximina Elnas Baylon
(Spouses At the time of their death, Spouses Baylon were survived by their legitimate children,
namely, Rita, Victoria, Dolores, Panfila, Ramon and herein petitioner Lilia B. Ada. Dolores died
intestate and without issue. Victoria died and was survived by her daughter, herein petitioner Luz
B. Adanza. Ramon died intestate and was survived by herein respondent Florante Baylon, his
child from his first marriage, as well as by petitioner Flora Baylon, his second wife, and their
legitimate children, namely, Ramon, Jr. and herein petitioners Remo, Jose, Eric, Florentino and
Ma. Ruby, all surnamed Baylon.
On July 3, 1996, the petitioners filed with the RTC a Complaint 4 for partition, accounting and
damages against Florante, Rita and Panfila. They alleged therein that Spouses Baylon, during
their lifetime, owned 43 parcels of land5 all situated in Negros Oriental. After the death of Spouses
Baylon, they claimed that Rita took possession of the said parcels of land and appropriated for
herself the income from the same. In their Answer, 8 Florante, Rita and Panfila asserted that they
and the petitioners co-owned 229 out of the 43 parcels of land mentioned in the latters complaint,
whereas Rita actually owned 10 parcels of land10 out of the 43 parcels which the petitioners sought
to partition, while the remaining 11 parcels of land are separately owned by Petra Cafino
Adanza,11 Florante,12 Meliton Adalia,13 Consorcia Adanza,14 Lilia15 and Santiago Mendez.
During the pendency of the case, Rita, through a Deed of Donation dated July 6, 1997, conveyed
Lot No. 4709 and half of Lot No. 4706 to Florante. On July 16, 2000, Rita died intestate and
without any issue. Thereafter, learning of the said donation inter vivos in favor of Florante, the

petitioners filed a Supplemental Pleading 17 dated February 6, 2002, praying that the said
donation in favor of the respondent be rescinded in accordance with Article 1381(4) of the Civil
Code. They further alleged that Rita was already sick and very weak when the said Deed of
Donation was supposedly executed and, thus, could not have validly given her consent thereto.
Florante and Panfila opposed the rescission of the said donation, asserting that Article 1381(4) of
the Civil Code applies only when there is already a prior judicial decree on who between the
contending parties actually owned the properties under litigation. 18
The RTC held that the death of Rita during the pendency of the case, having died intestate and
without any issue, had rendered the issue of ownership insofar as parcels of land which she claims
as her own moot since the parties below are the heirs to her estate. Thus, the RTC regarded Rita
as the owner of the said 10 parcels of land and, accordingly, directed that the same be partitioned
among her heirs. Nevertheless, the RTC rescinded the donation inter vivos of Lot No. 4709 and
half of Lot No. 4706 in favor of Florante. In rescinding the said donation inter vivos, Florante filed
a motion for reconsideration but denied.
On appeal, the CA reversed the Decision of the trial court. The CA held that before the petitioners
may file an action for rescission, they must first obtain a favorable judicial ruling that Lot No.
4709 and half of Lot No. 4706 actually belonged to the estate of Spouses Baylon and not to Rita.
Until then, the CA asserted, an action for rescission is premature. Further, the CA ruled that the
petitioners action for rescission cannot be joined with their action for partition, accounting and
damages through a mere supplemental pleading.
The petitioners sought reconsideration of the Decision dated October 26, 2007 but it was denied
by the CA.
Issue: Whether or not a supplemental pleading may raise a new cause of action as in the original
complain?
Held
Yes. A supplemental pleading may raise a new cause of action as long as it has some
relation to the original cause of action set forth in the original complaint.
Section 6, Rule 10 of the Rules of Court reads:
Sec. 6. Supplemental Pleadings. Upon motion of a party the court may, upon reasonable notice
and upon such terms as are just, permit him to serve a supplemental pleading setting forth
transactions, occurrences or events which have happened since the date of the pleading sought to
be supplemented. The adverse party may plead thereto within ten (10) days from notice of the
order admitting the supplemental pleading.
As its very name denotes, a supplemental pleading only serves to bolster or add something to the
primary pleading. A supplement exists side by side with the original. It does not replace that
which it supplements. Moreover, a supplemental pleading assumes that the original pleading is to
stand and that the issues joined with the original pleading remained an issue to be tried in the
action. It is but a continuation of the complaint. Its usual office is to set up new facts which justify,
enlarge or change the kind of relief with respect to the same subject matter as the controversy
referred to in the original complaint.
The purpose of the supplemental pleading is to bring into the records new facts which will enlarge
or change the kind of relief to which the plaintiff is entitled; hence, any supplemental facts which
further develop the original right of action, or extend to vary the relief, are available by way of

supplemental complaint even though they themselves constitute a right of action. 37 (Citations
omitted and emphasis ours)
Thus, a supplemental pleading may properly allege transactions, occurrences or events which had
transpired after the filing of the pleading sought to be supplemented, even if the said
supplemental facts constitute another cause of action.
the issue as to the validity of the donation inter vivos of Lot No. 4709 and half of Lot No. 4706
made by Rita in favor of Florante is a new cause of action that occurred after the filing of the
original complaint. However, the petitioners prayer for the rescission of the said donation inter
vivos in their supplemental pleading is germane to, and is in fact, intertwined with the cause of
action in the partition case. Lot No. 4709 and half of Lot No. 4706 are included among the
properties that were sought to be partitioned.
The petitioners supplemental pleading merely amplified the original cause of action, on account
of the gratuitous conveyance of Lot No. 4709 and half of Lot No. 4706 after the filing of the
original complaint and prayed for additional reliefs, i.e., rescission. Indeed, the petitioners claim
that the said lots form part of the estate of Spouses Baylon, but cannot be partitioned unless the
gratuitous conveyance of the same is rescinded. Thus, the principal issue raised by the petitioners
in their original complaint remained the same.
42. Barba vs. Liceo de Cagayan University
G.R. No. 193857 Nov. 28, 2012
PONENTE: Justice Villarama, Jr., J.
FACTS:
Petitioner Dr. Ma. Mercedes L. Barba (Barba) was the Dean of the College of Physical Therapy of
respondent Liceo de Cagayan University, Inc. (Liceo).
In the school year 2003 to 2004, the College of Physical Therapy suffered a dramatic decline in
the number of enrollees from a total of 1,121 students in the school year 1995 to 1996 to only 29
students in the first semester of school year 2003 to 2004. Due to the low number of enrollees,
the College was frozen indefinitely. Thereafter, Barba went on leave without pay starting.
Subsequently, Liceo sent Barba a letter dated April 27, 2005 instructing Barba to return to work
on and report to Ma. Chona Palomares, the Acting Dean of the College of Nursing, to receive her
teaching load and assignment as a full-time faculty member in that department. Barba did not
report to Palomares and requested for the processing of her separation benefits in view of the
closure of the College of Physical Therapy. Hence, Liceo sent Barba a notice terminating her
services on the ground of abandonment.
Barba filed a complaint before the Labor Arbiter for illegal dismissal, payment of separation pay
and retirement benefits against the University. She alleged that her transfer to the College of
Nursing as a faculty member is a demotion amounting to constructive dismissal.
The LA ruled that Barba was not constructively dismissed. The NLRC reversed the LA. Liceo went
to the CA and filed a Supplemental Petition raising for the first time the issue of lack of
jurisdiction of the Labor Arbiter and the NLRC over the case. Liceo claimed that a College Dean is
a corporate officer under its by-laws and Barba was a corporate officer of Liceo since her
appointment was approved by the board of directors. Thus, Liceo maintained that the jurisdiction
over the case is with the regular courts and not with the labor tribunals.

In its original Decision, the CA reversed the NLRC resolutions. The CA did not find merit in
Liceos assertion in its Supplemental Petition that the position of Barba as College Dean was a
corporate office. The CA further found that no constructive dismissal occurred nor has Barba
abandoned her work.
ISSUE:
Does the Labor Arbiter have jurisdiction?
Was there constructive dismissal?
RULING:
Yes. Labor tribunals have jurisdiction over Barbas complaint.
Corporate officers are elected or appointed by the directors or stockholders, and are those who are
given that character either by the Corporation Code or by the corporations by-laws. Section 25 of
the Corporation Code enumerates corporate officers as the president, the secretary, the treasurer
and such other officers as may be provided for in the by-laws. In Matling Industrial and
Commercial Corporation v. Coros, the phrase "such other officers as may be provided for in the
by-laws" has been clarified, thus: "Conformably with Section 25, a position must be expressly
mentioned in the By-Laws in order to be considered as a corporate office. The rest of the
corporate officers could be considered only as employees of subordinate officials.
However, an assiduous perusal of these documents does not convince us that Barba occupies a
corporate office position in the university. In Liceos by-laws, there are four officers specifically
mentioned, namely, a president, a vice president, a secretary and a treasurer. In addition, it is
provided that there shall be other appointive officials, a College Director and heads of
departments whose appointments, compensations, powers and duties shall be determined by the
board of directors. It is worthy to note that a College Dean is not among the corporate officers
mentioned in Liceos by-laws. Barba was not directly elected nor appointed by the board of
directors to any corporate office but her appointment was merely approved by the board together
with the other academic deans of respondent university in accordance with the procedure
prescribed in Liceos Administrative Manual. Though the board of directors may create appointive
positions other than the positions of corporate officers, the persons occupying such positions
cannot be deemed as corporate officers as contemplated by Section 25 of the Corporation Code.
Thus, petitioner, being an employee of respondent, her complaint for illegal/constructive
dismissal against respondent was properly within the jurisdiction of the Labor Arbiter and the
NLRC.
On the issue of constructive dismissal, there was none committed Barba was not constructively
dismissed.
Barbas letter of appointment specifically appointed her as Dean of the College of Physical Therapy
and Doctor-in-Charge of the Rehabilitation Clinic "for a period of three years effective July 1,
2002 unless sooner revoked for valid cause or causes." Evidently, Barbas appointment as College
Dean was for a fixed term, subject to reappointment and revocation or termination for a valid
cause. When Liceo decided to close its College of Physical Therapy due to drastic decrease in
enrollees,Barbas appointment as its College Dean was validly revoked and her subsequent
assignment to teach in the College of Nursing was justified as it is still related to her scholarship
studies in Physical Therapy. Particularly, for a transfer not to be considered a constructive
dismissal, the employer must be able to show that such transfer is not unreasonable,
inconvenient, or prejudicial to the employee.
43. Bautista vs. Maya-Maya Colleges, Inc.
476 SCRA 416

FACTS:

The petitioners are the spouses Rafael and Ligaya Bautista who are the owners of a 3,856 sq.
meter land in Batangas with OCT No. P-1436 under their name issued on January 15 th 1989. On
May 13, 1996, Respondent Maya Maya Colleges, Inc. filed a complaint with the RTC for
cancellation of their title and damages with application for preliminary injunction. They allege
that the petitioner were able to obtain the OTC through dubious means. On May 29, 1996, the
petitioners filed a motion to dismiss on the ground that it does not state a cause of action. Since
the respondent is a private corporation, the petitioner alleged that they are disqualified from
acquiring public alienable lands except by lease and therefore not considered a real party in
interest. The RTC granted the motion to dismiss. The respondent filed a motion for
reconsideration with motion for leave to file an amended complaint for quieting of title.
Respondent alleged that the technical description in petitioners title does not cover the disputed
lot. Petitioner filed an opposition claiming that the amended complaint still has no cause of
action.The RTC denied the petitioners motion to dismiss and reversed the order dismissing the
complaint. Petitioner filed with the Court of Appeals a special civil action for certiorari and
prohibition but the CA dismissed the petition. A motion for reconsideration was filed by the
petitioner but the same was denied by the CA hence, this petition.

ISSUE:

Whether or not the Court of Appeals erred in holding that the trial court did not commit grave
abuse of discretion amounting to lack or in excess of jurisdiction in admitting respondents
amended complaint.

RULING:

No. CA did not commit grave abuse of discretion amounting to lack or in excess of jurisdiction in
admitting respondents amended complaint.

Under Rule 10, Section 2 of the Rules of Civil Procedure, it states that A party may amend his
pleading once as a matter of right at any time before a responsive pleading is served or, in the case
of a reply, at any time within ten (10) days after it is served.

A party has an absolute right to amend a pleading when it is done before the filing of any
responsive pleading by the other party. A motion to dismiss is not considered as a responsive
pleading contemplated by the rule. Therefore, the respondent as a plaintiff may file an amended

complaint even after the original complaint was ordered dismissed, provided that the order of
dismissal is not yet final. Hence, the Court of Appeals did not gravely abuse its discretion. The
petition is denied and the decision and resolution of the Court of Appeals was affirmed in toto.

44. Siasoco vs. Court of Appeals (303 SCRA 186) Maling


45. PETLGC vs. Philippine Infrastructures, Inc.
G.R. No. 120384
January 13, 2004
AUSTRIA-MARTINEZ, J.:
Facts:
The case was commenced at the Regional Trial Court on October 30,
1986, upon the filing by herein petitioner of a complaint for collection of sum
of money against herein respondents Philippine Infrastructures, Inc. (PII for
brevity), Philippine British Assurance Co., Inc. (PBAC), The Solid Guaranty, Inc.
(Solid), B.F. Homes, Inc. (BF Homes), Pilar Development Corporation (PDC)
and Tomas B. Aguirre (Aguirre). The complaint alleges that: petitioner issued
five separate Letters of Guarantee in favor of the Philippine National Bank
(PNB) as security for various credit accommodations extended by PNB to
respondent PII; respondents PII, BF Homes, PDC and Aguirre executed a Deed
of Undertaking binding themselves, jointly and severally, to pay or reimburse
petitioner upon demand such amount of money or to repair the damages,
losses or penalties which petitioner may pay or suffer on account of its
guarantees; as security for prompt payment by respondent PII, the latter
submitted to petitioner, surety and performance bonds issued by respondents
PBAC and Solid;
PII refused to settle said obligations; petitioner likewise demanded
payment from respondents Solid and PBAC but they also refused to pay
petitioner; and because of the unjustified refusal of respondents to comply
with their respective obligations, petitioner was constrained to secure the
services of counsel and incur expenses for the purpose of prosecuting its
valid claims against the respondents.
A debit memo issued by the PNB, showing that the latter was paid by the
National Treasurer in behalf of petitioner corporation. Consequently, on
February 19, 1992, petitioner filed a Motion to Amend Complaint to Conform
to Evidence6 pursuant to Section 5, Rule 10 of the Revised Rules of Court,
seeking to amend pertinent portion in the complaint.
Acting on the motion to amend, the trial court, at that time presided by
Judge Joselito J. Dela Rosa, issued the assailed Order dated December 7,
1992, dismissing the case without prejudice on the ground of failure of the
complaint to state a cause of action, thus in effect, reversing the Order dated
June 10, 1987 issued by Judge Lagman five years earlier.
Issue: Whether or not the trial court erred in not allowing the amendment of
the complaint?
Ruling:

Yes, it was patently erroneous on the part of the trial court not to have
allowed the amendments as to make the complaint conform to petitioners
evidence that was presented without any objection from respondents. The
trial court likewise patently acted with grave abuse of discretion or in excess
of its jurisdiction amounting to lack of jurisdiction when, acting on a mere
motion to amend the complaint, it erroneously dismissed the complaint on
the ground of failure to state a cause of action. Consequently, the Court of
Appeals committed a reversible error in sustaining the trial court.
The trial court issued an order of dismissal instead of granting a motion
to amend complaint to conform to evidence, pursuant to Section 5, Rule 10 of
the Revised Rules of Court. It should be stressed that amendment was sought
after petitioner had already presented evidence, more specifically, the
testimony of petitioners Treasury Department Manager and a debit memo
from the PNB proving that petitioner had paid the PNB in the amount of
P19,035,256.57 pursuant to the guarantees it accorded to respondent PII.
A scrutiny of the pleadings filed by respondents reveal that none of
them denied petitioners claim that said evidence was presented before the
trial court without objections having been raised by respondents. None of
them claimed that they raised any objections at the time when petitioner
presented its evidence to prove its payment to PNB. Respondents Pilar and
Aguirre admitted the presentation of the said evidence.
The presentation of the contrariant evidence for and against
imputations undoubtedly cured, clarified or expanded, as the case may be,
whatever defects in the pleadings or vagueness in the issues there might
have been in the amended complaint. . . .
It is settled that even if the complaint be defective, but the parties go
to trial thereon, and the plaintiff, without objection, introduces sufficient
evidence to constitute the particular cause of action which it intended to
allege in the original complaint, and the defendant voluntarily produces
witnesses to meet the cause of action thus established, an issue is joined as
fully and as effectively as if it had been previously joined by the most perfect
pleadings. Likewise, when issues not raised by the pleadings are tried by
express or implied consent of the parties, they shall be treated in all respects
as if they had been raised in the pleadings.
Evidently, herein respondents failure to object to the evidence at the
time it is presented in court is fatal to their cause inasmuch as whatever
perceived defect the complaint had was cured by the introduction of
petitioners evidence proving actual loss sustained by petitioner due to
payment made by it to PNB.
46. SWAGMAN HOTELS AND TRAVEL, INC. VS. CA AND CHRISTIAN
G.R. No. 161135 April 8,2005
CHIEF JUSTICE DAVIDE, JR.
FACTS:
Petitioner Swagman Hotels and Travel, Inc., through Atty. Leonor Infante and Rodney Hegerty,
obtained from respondent Neal Christian loans evidenced by three promissory notes dated 7

August 1996, 14 March 1997, and 14 July 1997. Each loan was in the amount of USD 50,000
which is payable after three years from its date and with an interest of 15% per annum payable in
three months. However, on 16 December 1999, private respondent decided to terminate the loans
and demanded petitioner to pay the loaned amount of USD 150,000 and interest amounting to
USD 13,500.
Christian filed a complaint for sum of money and damages with the Regional Trial Court of
Baguio against petitioners on 2 February 1999. In its defense, petitioners claimed that there was
no cause of action since the payment of the loans were not yet due and demandable at the time the
complaint was filed. The RTC decided in favor of private respondent, on the ground that although
the payments were not yet due and demandable at the time of the filing of the complaint, these
have already become due and demandable at the time the Decision was made. The complaint may
be cured by an amendment with leave of court. The Court of Appeals affirmed in toto the Decision
of the RTC.
ISSUE:
Whether or not the amendment of the complaint will cure the defect of lack of cause of action of
the complaint.
HELD:
It was held by the Supreme Court that an amendment of the complaint will not cure its defect. It
was emphasized by the Supreme Court that there was no cause of action when the complaint was
filed since none of the loans were due and demandable. Section 5, Rule 10 of the 1997 Rules of
Court is not applicable to this case because said provision is applicable only when there is a cause
of action when the complaint is filed.
Also, a complaint whose cause of action has not yet accrued cannot be cured by an amended or
supplemental pleading alleging existence or accrual of a cause of action while the case is pending.
It is considered prematurely files and is, therefore, a groundless suit.
The case was dismissed.
47. VICTORIA J. ILANO represented by her Attorney-in-fact, MILO ANTONIO C.
ILANO vs. HON. DOLORES L. ESPANOL, in her capacity as Executive Judge, RTC
of Imus, Cavite, Br. 90, and, AMELIA ALONZO, EDITH CALILAP, DANILO
CAMACLANG, ESTELA CAMACLANG, ALLAN CAMACLANG, LENIZA REYES,
EDWIN REYES, JANE BACAREL, CHERRY CAMACLANG, FLORA CABRERA,
ESTELITA LEGASPI, CARMENCITA GONZALES, NEMIA CASTRO, GLORIA
DOMINGUEZ, ANNILYN C. SABALE and several JOHN DOES.
G.R. No. 161756,

December 16,2005

CARPIO MORALES, J.
Facts:
Respondents by means of deceit and abuse of confidence succeeded in procuring Promissory
Note and signed blank checks from petitioner who then recuperating from illness, especially

in those times when petitioner left for the United States for medical check-up .
Petitioner filed a complaint for Revocation/Cancellation of Promissory Note and Bills of Exchange
against respondents before the RTC. Respondents filed their respective Answers

invoking, among other grounds for dismissal, lack of cause of action, for while
the checks subject of the complaint had been issued on account and for value,
some had been dishonored due to account closed and the allegations in the
complaint are bare and general. The trial court dismissed petitioners
complaint for failure to allege the ultimate facts.
Issue:
Whether or not the dismissal is proper?
Held:

No. Where the allegations of a complaint are vague, indefinite, or in the


form of conclusions, its dismissal is not proper for the defendant may ask for
more particulars. In this case, even if some are not stated with particularity,
petitioner alleged 1) her legal right not to be bound by the instruments which
were bereft of consideration and to which her consent was vitiated; 2) the
correlative obligation on the part of the defendants-respondents to respect
said right; and 3) the act of the defendants-respondents in procuring her
signature on the instruments through deceit, abuse of confidence
machination, fraud, falsification, forgery, defraudation, and bad faith, and with
malice, malevolence and selfish intent.

48 Republic vs Sandiganbayan
GR No. 152375 December 16,2011
Brion, J.:

FACTS:
Petitioner, the Republic of the Philippines, through the Presidential Commission on Good
Governance (PCGG) filed a complaint against the respondents and was docketed as Civil Case No.
0009 for reconveyance, reversion, accounting, restitution, and damages before the
Sandiganbayan. Petitioner alleged that the respondents illegally manipulated the purchase of the
major shareholdings of Cable and Wireless limited in Eastern Telecommunications Philippines,
Inc. (ETPI) which respondents Jose Africa and Manuel Nieto Jr. held for themselves and through
their holdings and the corporations they organized, beneficially for respondents Ferdinand E.
Marcos and Imelda R. Marcos.
Civil Case No. 0009 is the main case subject of the present petition. Victor Africa (Africa),
son of the late Jose L. Africa, was not impleaded in and so is plainly not a party to Civil Case No.
0009. Civil Case No. 0009 spawned numerous incidental cases, among them, Civil Case No.
0130. The present respondents were not made parties either in Civil Case No. 0130.
Sandiganbayan ordered the consolidation of the two cases.
In the proceedings to resolve the Urgent Petition, the testimony of one Maurice Bane
(former director and treasurer-in-trust of ETPI) was taken at the petitioners instance and after
serving notice of the deposition-taking on the respondents by way of deposition upon oral
examination (Bane deposition) before Consul General Ernesto Castro of the Philippine Embassy
in London, England.

ISSUE:
WON the Sandiganbayan committed grave abuse of discretion
WON the Bane Deposition is admissible as evidence.
HELD:

Petition is dismissed for lack of merit.


The Supreme Court Held that: (1) the Sandiganbayans denial of the
petitioners 3rd motion the Motion to Admit Supplemental Offer of Evidence
(Re: Deposition of Maurice Bane) was a legal error that did not amount to
grave abuse of discretion; (2) the Sandiganbayans refusal to reopen the case
at the petitioners instance was tainted with grave abuse of discretion;
and (3) notwithstanding the grave abuse of discretion, the petition must
ultimately fail as the Bane deposition is not admissible under the rules of
evidence.

49. Virata vs. Sandiganbayan (202 SCRA 680) Pangan


50. Republic vs. Cagioua (691 SCRA 306)
Facts:
Indigo Distribution Corporation and thirteen other petitioners filed before the respondent judge
a petition for declaratory relief with prayer for temporary restraining order (TRO) and
preliminary mandatory injunction against the Honorable Secretary of Finance, et al seeking to
nullify the implementation of Section 6 of Republic Act (R.A.) No. 9334 wherein it states that,
The provision of any special or general law to the contrary notwithstanding, the importation of
cigars and cigarettes, distilled spirits, fermented liquors and wines into the Philippines, even if
destined for tax and duty-free shops, shall be subject to all applicable taxes, duties, charges,
including excise taxes due thereon
The petitioners in this case are importers and traders duly licensed to operate inside the Subic
Special Economic and Freeport Zone (SSEFZ) and was granted Certificates of Registration and
Tax Exemption from their importations. The TRO was granted. The Republic filed before this
Court a petition for certiorari and prohibition to annul that order and also asked the judge to
suspend the proceedings pending its resolution. The private respondents (in the present petition
now before us) filed motions for leave to intervene and to admit its complaints-in-intervention
which was granted because he found that it was too adversely affected by the contested Act.
Republic contends that the respondent judge violated its right to due process when he
peremptorily allowed the private respondents motions and complaints-in-intervention and
proceeded with their hearing ex parte despite the absence of any prior notice to it. The Republic
maintains that it never received any notice of hearing, nor any copy of the questioned motions
and complaints-in-intervention.
The respondent judge on the other hand, denied Republics MR and held that all of the parties
in the original case had been duly notified as shown by the records
Issue:
Whether or not there was grave abuse of discretion on the part of respondent judge when it
allowed the private respondents motions and complaints-in-intervention and proceeded with
their hearing ex parte despite the absence of any prior notice to it.
Ruling:
NO. Under our rules of procedure, service of the petition on a party, when that party is
represented by a counsel of record, is a patent nullity and is not binding upon the party
wrongfully served. This rule, however, is a procedural standard that may admit of exceptions
when faced with compelling reasons of substantive justice manifest in the petition and in the
surrounding circumstances of the case. Procedural rules can bow to substantive considerations
through a liberal construction aimed at promoting their objective of securing a just, speedy and
inexpensive disposition of every action and proceeding.
The Republic has consistently and repeatedly maintained that it never received a copy of the
motions and complaints-in-intervention, as evidenced by the certification of the Docket Division

of the Office of the Solicitor General (OSG); it learned of the private respondents presence in this
case only after it received copies of the assailed orders, and it even had to inquire from the lower
court for the private respondents addresses. Although their counsels (of private
respondents) did not formally receive any copy of the petition, the private
respondents themselves admitted that they received their copy of the present
petition. The records show that the Republic subsequently complied with the rules
on service when, after the private respondents comment, the Republic served
copies of its reply and memorandum to the respondents counsel of record.
Under these circumstances, we are satisfied with the Republics explanation on why it failed to
initially comply with the rule on service of the present petition; its subsequent compliance
with the rule after being informed of the presence of counsels of record sufficiently
warrants the rules relaxed application. The lack of a proper serviceunlike the situation
when the Republic was simply confronted with already-admitted complaints-in-interventiondid
not result in any prejudice; the private respondents themselves were actually served with, and
duly received, their copies of the present petition, allowing them to comment and to be heard on
the petition.

51. UNIVERSITY OF THE EAST, DEAN ELEANOR JAVIER, RONNIE GILLEGO and
DR. JOSE C. BENEDICTO, Petitioners,
vs.
ANALIZA F. PEPANIO and MARITI D. BUENO, Respondents.
G.R. No. 193897

January 23, 2013

Facts:
In 1992, the Department of Education, Culture and Sports (DECS) issued the Revised
Manual of Regulations for Private Schools, Article IX, Section 44, paragraph 1 (a), of which
requires college faculty members to have a master's degree as a minimum educational
qualification for acquiring regular status.
Meantime, on February 7, 1996 several concerned government agencies issued DECSCHED-TESDA-DOLE Joint Order 1 which reiterated the policy embodied in the Manual of
Regulations that "teaching or academic personnel who do not meet the minimum academic
qualifications shall not acquire tenure or regular status." In consonance with this, the UE
President issued a University Policy stating that, beginning the School Year 1996-1997, it would
hire those who have no postgraduate units or masters degree for its college teaching staffs, in the
absence of qualified applicants, only on a semester-to-semester basis.
UE hired respondent Mariti D. Bueno in 1997 and respondent Analiza F. Pepanio in
2000, both on a semester-to-semester basis to teach in its college. They could not qualify for
probationary or regular status because they lacked postgraduate degrees.
In 2001 UE and the UE Faculty Association entered into a new CBA that would have the
school extend probationary full-time appointments to full-time faculty members who did not yet
have the required postgraduate degrees provided that the latter comply with such requirement
within their probationary period.

Pursuant to the new CBA, UE extended probationary appointments to respondents Bueno


and Pepanio. Then again, Respondent Pepanio requested a three-semester extension but Dean
Javier denied this request and directed Pepanio to ask for just a two-semester extension. The
records do not show if Bueno submitted a request for extension. At any rate, the school eventually
wrote respondents, extending their probationary period but neither Pepanio nor Bueno reported
for work.
Bueno later wrote UE, demanding that it consider her a regular employee based on her
six-and-a-half-year service on a full-load basis, given that UE hired her in 1997 when what was in
force was still the 1994 CBA. Pepanio made the same demand, citing her three-and-a-half years of
service on a full-load basis. When UE did not heed their demands, respondents filed cases of
illegal dismissal against the school before the Labor Arbiter s (LA) office.
For its defense, UE countered that it never regarded respondents as regular employees since they
did not hold the required masters degree that government rules required as minimum
educational qualification for their kind of work. On March 10, 2005 the LA held that Bueno and
Pepanio were regular employees.
Dissatisfied, UE appealed to the National Labor Relations Commission (NLRC).Bueno
and Pepanio questioned the timeliness of the appeal to the NLRC. They pointed to the
postmasters certification that its office received the mail containing the LAs Decision on March
17, 2005 and "informed the Office of Atty. Mison right away but they only got the letter on April 4,
2005." Bueno and Pepanio claim that the 10-day period for appeal should be counted from March
22, 2005, five days after the postmasters first notice to Atty. Mison to claim his mail. On
September 27, 2006 the NLRC Third Division set aside the LA Decision.1 It rejected the technical
objection and ruled that the four-semester probationary period provided under the old CBA did
not automatically confer permanent status to Bueno and Pepanio
On petition for certiorari, the Court of Appeals (CA) rendered a Decisionon July 9, 2010,
reinstating the LAs Decision by reason of technicality. It held that the 10-day period for appeal
already lapsed when UE filed it on April 14, 2005 since the reckoning period should be counted
five days from March 17, when the postmaster gave notice to UEs legal counsel to claim his mail
or from March 22, 2005. This prompted UE to file the present petition.
Issue:
Whether or not UE filed a timely appeal to the NLRC from the Decision of the LA
Ruling:
For completeness of service by registered mail, the reckoning period starts either (a) from
the date of actual receipt of the mail by the addressee or (b) after five days from the date he
received the first notice from the postmaster. There must be a conclusive proof, however, that the
registry notice was received by or at least served on the addressee before the five-day period
begins to run.
Here, the records fail to show that Atty. Mison in fact received the alleged registry notice
from the post office on March 22, 2005 that required him to claim his mail. Respondents have not
presented a copy of the receipt evidencing that notice. The Court has no choice but to consider the
registry return receipt bearing the date April 4, 2005 which showed the date of Atty. Misons
receipt of a copy of the LA Decision a conclusive proof of service on that date. Reckoned from
April 4, UE filed its appeal to the NLRC on time.

52. Planters Development Bank vs. Spouses Ernesto Lopez and Florentino Lopez,
substituted by Joseph Wilfred Joven, Joseph Gilbert Joven and Marlyn Joven
G.R. No. 186332 October 23, 2013
Brion, J.;
FACTS:
On May 18, 1983, spouses Ernesto and Florentino Lopez obtained a real estate loan in the
amount of P3,000,000.00 from Planters Bank. The loan was intended to finance the construction
of a four-story concrete dormitory building. The loan agreement was amended by the parties
three times. The third amendment to the loan agreement increased the amount of the loan to
P4,200,000.00 and the interest rate to 27% per annum.
The spouses Lopez failed to avail the full amount of the loan because Planters Bank
refused to release the remaining amount of P700,000.00. They filed a complaint for rescission of
the loan agreements and for damages with the Regional Trial Court (RTC) of Makati. The RTC
ruled in favor of Planters Bank. It held that the spouses Lopez had no right to rescind the loan
agreements because they were not the injured parties. The Court of Appeals (CA) reversed the
RTC ruling. It held that the refusal of Planters Bank to release the loan was a substantial breach
of the contract.
The respondents filed a motion for reconsideration seeking for clarification on the
dispositive portion which did not declare the rescission of the loan and accessory contracts. On
the other hand, Planters Bank filed a Comment of March 2, 2007 praying for the reinstatement of
the RTCs ruling. The CA re-examined the case and treated the Comment as a motion for
reconsideration. It affirmed its previous decision and modified the dispositive portion.
ISSUE:
Whether or not Planters Bank received the CAs amended decision on August 7, 2007 and
not on August 2, 2007 as evidenced by the certifications from the Manila and Makati Central Post
Offices
HELD:
Planters Bank received the CAs amended decision on August 7, 2007.
Section 13, Rule 13 of the Rules of Court provides that if service is made by registered
mail, proof shall be made by an affidavit of the person mailing it and the registry receipt issued by
the mailing office. However, the presentation of an affidavit and a registry receipt is not
indispensable in proving service by registered mail. Other competent evidence, such as the
certifications from the Philippine Post Office, may establish the fact and date of actual service.
These certifications are direct and primary pieces of evidence of completion of service.
The certifications from the Manila Central Post Office and Makati Central Post Office
presented by Planters Bank conclusively show that their counsel received the amended decision
on August 7, 2007 and not on August 2, 2007. The Manila Central Post Office certification states

the amended decision was only dispatched from the ManilaCentral Post Office
to the Makati Central Post Office on August 2, 2007. On the other hand, the
Makati Central Post Offices certification provides that Planters Banks actual
receipt of the decision was on August 7, 2007. Consequently, the motion for
reconsideration dated August 22, 2007 was filed by Planters Bank on time.
53. Rogelio Aberca, et. al. vs. Maj. Gen. Fabian Ver et. Al.
G.R. No. 166216

March 14, 2012

Mendoza, J.
Facts:
On 25 January 1983, several suspected subversives who were arrested and
detained by the military filed a complaint for damages with the Regional Trial
Court of Quezon City against Gen. Fabian Ver, then AFP Chief of Staff, and the
following subordinate officers: Col. Fidel Singson, Col. Gerardo Lantoria, Col.
Rolando Abadilla, Col. Guillermo Kintanar, Lt. Col. Panfilo Lacson, Maj. Rodolfo
Aguinaldo, Capt. Danilo Pizarro, 1Lt. Pedro Tango, 1Lt. Romeo Ricardo, 1Lt.
Raul Bacalso, M/Sgt. Bienvenido Balaba and "John Does."
Due to certain events (EDSA Revolution, Burning of the QC Hall of Justice) the
records of the case were lost by burning. Plaintiffs then filed a petition for the
reconstitution of the case which was set for hearing on October 27, 1989.
However, nothing in the records show that defendants-apellants were
notified. Due to lack of opposition, the petition for reconstitution was granted
in an order dated March 12, 1990.
Thereafter, Plaintiffs were required to report to the court to report the
addresses and whereabouts of the defendants so that the latter may be
notified. However, instead of complying, plaintiffs move for the declaration of
the defendants in default but was denied.
For failure to comply with the order of the court to report the addresses and
whereabouts of the defendants, the case was dismissed but was
subsequently reinstated. Plaintiffs moved that the notice to file answer of
responsive pleading thru publication which was granted by the court.
Thereafter, the notice was published in BALITA newspaper in August 29,1991
and September 5, 1991.
Issue:
Whether or Not there was valid service of the notice to file responsive
pleading to the defendants.
Ruling:

No, there was no valid service of the notice to the defendants.


Nothing in Section 5 of Rule 13 of the then rules of court provides for the
service thru publication. It only provided for the following modes: (1)
Personal; (2) Registered Mail; and (3) Substituted Service only. In this case,
there was no showing that any of the above means were used to serve the
notice to the defendants thus, it violated their right to due process
guaranteed under Article III, Section 1 of the Constitution.
Therefore, the courts decision imposing liabilities on the defendants was
reversed and the case was accordingly dismissed.
54. CITY OF DUMAGUETE, herein Represented by City Mayor, Agustin
R. Perdices, Petitioner.
Vs.
PHILIPPINE PORTS AUTHORITY, Respondent.
LEONARDO-DE CASTRO, J.
SERVICE OF PLEADINGS
Factual antecedents:

On October 14, 1998, petitioner City of Dumaguete, through Mayor


Felipe Antonio B. Remollo (Remollo), filed before the RTC an Application
for Original Registration of Title over a parcel of land with
improvements, located at Barangay Looc, City of Dumaguete (subject
property), under the Property Registration Decree.
The Republic of the Philippines, represented by the Director of Lands,
and respondent, represented by the Office of the Government
Corporate Counsel, filed separate Oppositions to the application for
registration of petitioner.
Both the Republic and respondent averred that petitioner may not
register the subject property in its name since petitioner had never
been in open, continuous, exclusive, and notorious possession of the
said property for at least 30 years immediately preceding the filing of
the application; and the subject property remains to be a portion of the
public domain which belongs to the Republic.
After several postponements of the scheduled hearings, petitioner
presented the testimony of its first witness, Engineer Rilthe P. Dorado
(Engr. Dorado), on January 14, 2000.

However, before the next hearing, respondent filed a Motion to


Dismiss, seeking the dismissal of LRC Case No. N-201 on the ground
that the RTC lacked jurisdiction to hear and decide the
case. Respondent argued that Section 14(1) of Presidential Decree No.
1529, otherwise known as the Property Registration Decree, refers only
to alienable and disposable lands of the public domain under a bona
fide claim of ownership. The subject property in LRC Case No. N-201 is
not alienable and disposable, since it is a foreshore land, as explicitly
testified to by petitioners own witness, Engr. Dorado.

Respondent filed a Reply/Rejoinder (To Applicants Opposition to


Oppositors Motion to Dismiss), asserting that there are no factual or
legal basis for the claim of petitioner that the subject property is
reclaimed land.

In its Motion for Reconsideration and Supplemental Motion for


Reconsideration, petitioner contended that the dismissal of its
application was premature and tantamount to a denial of its right to
due process. Petitioner insisted that the RTC should continue with the
hearing of LRC Case No. N-201 and allow petitioner to present evidence
that the subject property is reclaimed land.

Respondent based its Opposition (To Applicants Motion for


Reconsideration dated September 28, 2000) and Opposition (To
Applicants Supplemental Motion for Reconsideration) on technical and
substantive grounds. Sections 4, 5, and 6, Rule 15 and Section 11, Rule
13 of the Rules of Court.

Issue: Error of Law: The March 4, 2005 decision of the Court of Appeals
and its June 6, 2005 Resolution, erred on question of law in setting
aside the Orders of the Regional Trial Court, Branch 44, dated
December 7, 2000 and February 20, 2001.
Held:
The Court of Appeals erred in granting the writ of certiorari in favor of
respondent. The RTC did not commit grave abuse of discretion when, in its
Orders dated December 7, 2000 and February 20, 2001, it set aside the order
of dismissal of LRC Case No. N-201 and resolved to have a full-blown
proceeding to determine factual issues in said case

And also, the Motion for Reconsideration and Supplemental Motion for
Reconsideration of the Order dated September 7, 2000 filed by petitioner did
not comply with Section 11, Rule 13 of the Rules of Court, for these did not
include a written explanation why service or filing thereof was not done
personally.
Counsel for petitioner holds office in Dumaguete City, Negros Oriental, in the
Visayas; while counsel for respondent holds office in Quezon City, Metro Manila,
in Luzon. Given the considerable distance between the offices of these two
counsels, personal service of pleadings and motions by one upon the other
was clearly not practicable and a written explanation as to why personal
service was not done would only be superfluous.
Jurisprudence confirms that the requirements laid down in Sections 4, 5, and
6, Rule 15 of the Rules of Court that the notice of hearing shall be directed to
the parties concerned, and shall state the time and place for the hearing of
the motion, are mandatory. If not religiously complied with, they render the
motion pro forma. As such, the motion is a useless piece of paper that will not
toll the running of the prescriptive period.
It was not refuted that petitioner furnished respondent and respondent
actually received copies of the Motion for Reconsideration, as well as the
Supplemental Motion for Reconsideration of the RTC Order dated September
7, 2000 filed by petitioner. As a result, respondent was able to file its
Oppositions to the said Motions. The RTC, in issuing its Order dated December
7, 2000, was able to consider the arguments presented by both sides. Hence,
there was substantial compliance by petitioner with the rules on notice of
hearing for its Motion for Reconsideration and Supplemental Motion for
Reconsideration of the RTC Order dated September 7, 2000. Respondent
cannot claim that it was deprived of the opportunity to be heard on its
opposition to said Motions.
55. EDITO PAGADORA vs JULIETA ILAO
G.R. No. 165769
PERALTA, J.:

FACTS: Respondent Julieta Ilao acquired, under a Contract to Sell, a 5,148 sq m piece of land in
Burgos, Rodriguez, Rizal. The contract stipulated that the balance of the purchase price was
payable upon proof by the vendee that the boundaries of the property had already been relocated
and that the fence thereon had been constructed. Immediately after the sale, respondent as
vendee had commissioned the survey of the property, but the work had been stalled because, on
several occasions the occupant of the adjoining lot, herein petitioner Edito Pagadora, had
allegedly prevented the surveyor from completing the task. Hence, respondent filed a Complaint
for forcible entry against petitioner before the MeTC, Rodriguez, Rizal.
ISSUE: Whether or not service through registered mail is proper in lieu of personal service

HELD: Service of pleading through registered mail is proper in lieu of personal service. A liberal
application of procedural rules requires that: there is justifiable cause or plausible explanation for
non-compliance and there is compelling reason to convince the court that the outright dismissal
would seriously impair or defeat the administration of justice. It is settled that liberal
construction of the rules may be invoked in situations where there may be some excusable formal
deficiency or error in a pleading, provided that the same does not subvert the essence of the
proceeding and it at least connotes a reasonable attempt at compliance with the rules. They ought
to be relaxed when there is subsequent or even substantial compliance, consistent with the policy
of liberality espoused by Rule 1, Section 6.

56. G.R. No. 172458


December 14, 2011
PHILIPPINE NATIONAL BANK, Petitioner, vs COMMISSIONER OF INTERNAL
REVENUE, Respondent.
FACTS:
In 1999, PNB filed with the Bureau of Internal Revenue (BIR) its Tax Returns for 1998 with
attached pertinent documents. However, in view of the payments made by PNB in excess of its
income tax liability for 1998, it filed with the Second Division of the Court of Tax Appeals (CTA
Division) a Petition for Review to recover the amount of 6,028,594.00 from BIR.
During the pendency of the case, the BIR refunded to PNB the amount of 4,154,353.42, leaving a
balance of 1,874,240.58 out of PNBs total claim. In 2005, the CTA Division rendered a Decision
ordering BIR to refund the amount of 1,428,661.66 ruling that the payment of the balance
amounting to 445,578.92 was not supported by pertinent documents. PNB filed a Motion for
Partial Reconsideration, asserting its entitlement to be refunded the amount of 445,578.92. The
CTA denied its motion.
Aggrieved, PNB filed a partial appeal by way of Petition for Review before the CTA En Banc. This
petition was received by the CTA En Banc on December 27, 2005, four days beyond the
reglementary period of 15 days for the timely filing of such petition. Hence, the CTA En Banc
denied the petition.
ISSUE:
Whether or not this Court should require the CTA En Banc to give due course to the Petition for
Review despite PNBs failure to comply with the formal requirements of the Revised Rules of the
Court of Tax Appeals and the Rules of Court.
HELD:
NO. The Court stressed that PNB failed to comply with not just one, but three procedural rules
when it filed its petition for review with the CTA En Banc.
PNB filed its petition via LB service which is a private messengerial service instead of registered
mail with return card. It has been established that a pleading filed by ordinary mail or by private
messengerial service is deemed filed on the day it is actually received by the court, and not on the
day it was mailed or delivered to the messengerial service. Also, service by ordinary mail is
allowed only in instances where no registry service exists as stipulated under Rule 13, Section 7 of
the Rules of Court. Since the CTA En Banc received the petition of Dec. 27, such pleading is
deemed filed on that date. Furthermore, the failure of PNB in attaching the required duplicate
originals or certified true copies of the decision and resolution being assailed, and the Affidavit of
Service as mandated by CTA Revised Rules warrants the dismissal of the petition.

57. Ellice Agro-Industrial Corp. vs. Rodel T. Young, et. Al.


G.R. No. 174077

November 21, 2012

Mendoza, J.
Facts:
On November 8, 1996, herein respondents filed a complaint for Specific
Performance against petitioner, Ellice Agro-Industrial Corporation and Atty.
Guia G. Domingo, its alleged corporate secretary and attorney0in-fact.
The Initial service of summons together with a copy of the complaint and its
annexes were served at EAIC, thru Domingo on Rizal Street Sariaya, Quezon
but was unsuccessful since EAIC could not be found on the said address.
The second was done at 966 Maligaya Street Singalong, Manila, the residence
of Domingo which was successful.
EAIC failed to appear in the pre-trial, thus, respondents presented evidence
ex-parte. Respondents won their case and no motion for reconsideration was
filed.
After 7 months, EAIC represented by Gala filed a Petition for Relief from
Judgment contending that, there was no calid service of summons to the
corporation.
Issue:
Whether or Not the RTC validly acquired jurisdiction over the person of EAIC.
Ruling:
No, the RTC did not acquire jurisdiction over the person of EAIC.
Section 13, Rule 14 of the 1964 Rules of Civil procedure provides that, Service
of Summons upon a Private Domestic Corporation, service must be made on
the President, Manager, Secretary, Cashier, Agent or any of its directors.
Nowhere in the General Information System of EIAC (on record with the
Securities and Exchange Commission) show that Domingo holds any of the
position stated in the rules. Thus, the Supreme Court is constrained to rule
that there was no valid service of summons on EAIC.
58. PUA VS DEYTO
GR NO. 173336 November 26, 2012

PONENTE: Justice Brion


FACTS:
Pua is engaged in the business of wholesale rice trading. Ang, one of his clients is operating
under the business of JD Grains Center. In 2002, Pua delivered Ang truckloads of rice worth
Php766, 800.00. ang paid 2 postdated checks but were dishonored because they were drawn from
a closed account. Pua then filed a complaint with the RTC for collection of sum of money with
preliminary attachment against Ang and Deyto, as co-owners of JD Grains which was granted.
Summons was duly served to Deyto but not on Ang who had absconded. Pua then moved for leave
of court to serve summons by publication on Ang which was granted. His counsel manifested that
the summons for Ang remained unpublished. Summons then was published in 2002. However in
2003, 6 months after publication, the case was archived since Pua neither filed any motions.
Hence, RTC dismissed the complaint. Pua appealed with the Ca which was denied.
ISSUE: Whether or not Pua incurred unreasonable delay in prosecuting the case.
RULING: Yes.
Sec. 14, Rule 14 of the Rules of Court provides:
Sec. 14. Service upon defendant whose identity or whereabouts are unknown. In any action
where the defendant is designated as an unknown owner, or the like, or whenever his
whereabouts are unknown and cannot be ascertained by diligent inquiry, service may, by leave of
court be effected upon him by leave of court, be effected upon him by publication in a newspaper
of general circulation and in such places and for such time as the court may order.
Until the summons has been served on ANg, the case cannot proceed since Ang is an
indispensable party to the case. An indispensable party is one who must be included in an action
before it may properly go forward. A court must acquire jurisdiction over the person of
indispensable parties before it can validly pronounce judgments personal to parties.

59. SIXTO N. CHU, Petitioner, v. MACH ASIA TRADING CORPORATION,


Respondent.
G.R. No. 184333
April 1, 2013
PERALTA, J
FACTS:
Respondent Mach Asia Trading Corporation is a corporation engaged in importing dump
trucks and heavy equipments. Petitioner Sixto N. Chu purchased on installment one Hitachi
Excavator worth P900,000.00 from the respondent. Petitioner initially paid P180,000.00 with
the balance of P720,000.00 to be paid in 12 monthly installments through Prime Bank postdated
checks. Petitioner again purchased two heavy equipments from the respondent on installment
basis in the sum of P1,000,000.00, namely: one motorgrader and one payloader. Petitioner made
a down payment of P200,000.00 with the balance of P800,000.00 payable in 12 monthly
installments through Land Bank postdated checks. However, upon presentment of the checks for
encashment, they were dishonored by the bank either by reason of "closed account," "drawn
against insufficient funds," or "payment stopped." Respondent informed petitioner that the

checks were dishonored and invited him to its office to replace the checks. Respondent sent
petitioner a formal demand letter urging the latter to settle his accounts within five days from
receipt of the letter. In response, petitioner sent respondent a letter explaining that his business
was badly hit by the Asian economic crisis and that he shall endeavor to pay his obligation by
giving partial payments.
Respondent filed a complaint before the Regional Trial Court of Cebu City for sum of
money, replevin, attorneys fees and damages against the petitioner. Respondent prayed for the
payment of the unpaid balance of P1,661,947.27 at 21% per annum until full payment, 25% of the
total amount to be recovered as attorneys fees, litigation expenses and costs. The RTC issued an
Order6 allowing the issuance of a writ of replevin on the subject heavy equipments. Sheriff
Doroteo P. Cortes proceeded at petitioners given address for the purpose of serving the
summons, together with the complaint, writ of replevin and bond. However, the Sheriff failed to
serve the summons personally upon the petitioner, since the latter was not there. The Sheriff then
resorted to substituted service by having the summons and the complaint received by a certain
Rolando Bonayon, a security guard of the petitioner. Petitioner failed to file any responsive
pleading, which prompted respondent to move for the declaration of defendant in default. The
RTC issued an Order declaring defendant in default and, thereafter, allowed respondent to
present its evidence ex parte. The RTC rendered a Decision against the petitioner. Petitioner
argued that the RTC erred in concluding that the substituted service of summons was valid, and
that, consequently, there was error on the part of the RTC when it declared him in default, in
proceeding with the trial of the case, and rendering an unfavorable judgment against him. The CA
rendered a Decision affirming the Decision of the RTC.
ISSUE: Whether there was no valid substituted service of summons.
HELD:
Yes, there was no valid substituted service of summons. Courts acquire jurisdiction over the
plaintiffs upon the filing of the complaint. On the other hand, jurisdiction over the defendants in a
civil case is acquired either through the service of summons upon them or through their voluntary
appearance in court and their submission to its authority. As a rule, summons should be
personally served on the defendant. It is only when summons cannot be served personally within
a reasonable period of time that substituted service may be resorted to. It is to be noted that in
case of substituted service, there should be a report indicating that the person who received the
summons in the defendant's behalf was one with whom the defendant had a relation of
confidence, ensuring that the latter would actually receive the summons.
In this case, it was not shown that the security guard who received the summons in behalf of
the petitioner was authorized and possessed a relation of confidence that petitioner would
definitely receive the summons. This is not the kind of service contemplated by law. Thus, service
on the security guard could not be considered as substantial compliance with the requirements of
substituted service.

#60 Macasaet v Co
G.R. No. 156759
Bersamin, J.:

June 5, 2013

Facts:
Respondent Francisco Co, Jr. is a retired police officer who sued Abante Tonite, a daily
tabloid of general circulation, together with its publisher, managing director, circulation manager,
editor, as well as columnist (Allen Macasaet, et al.) claiming damages for an allegedly libelous
article that they published. The RTC the issued summons to be served by its Sheriff Raul Medina.
In September 13, 2000 Sheriff Medina proceeded to the address of Macasaet, et al. to effect
the summons. His efforts to personally serve the summons proved futile however since the
petitioners were out of the office and unavailable. He returned in the afternoon of the same day
but was informed that they were still out of the office. He was then told by several persons that the
petitioners are constantly out and unavailable, thus prompting Sheriff Medina to resort to
substituted service instead. In his sheriffs return, he explained the various reasons which led him
to believe that he had to resort to substituted service, namely: Macasaets secretary informed him
that Macasaet was always out and unavailable, Quijanos wife said he was always out and
unavailable, Rene Esleta, an Editorial assistant of Abante Tonite said that the other petitioners
Albano, Bay, Galang, Hagos and Reyes were always out gathering news.
On October 3, 2000, the petitioners moved for the dismissal of the complaint alleging the
lack of jurisdiction over their persons because of the substituted summons. They contended that
no prior attempt was made to serve the summons as required by Sections 6 and 7 of Rule 14 of the
Rules of Court.

Issue:
WON there was valid service of the summons, allowing the court to acquire jurisdiction over
the petitioners.

Held:
YES. There is no question that Sheriff Medina attempted to serve the summons upon each of
the petitioners in person twice. After Medina learned from those present in the office that it would
be quite impossible to reach the petitioners during the business hours of that day or any day, his
conclusion that it would be best to resort to substituted service is warranted. Reasonable time for
serving summons is dependent on the circumstances obtaining. It is the spirit of procedural rules,
not their letter, which governs.
In any case, the petitioners claim and insistence on personal service is clealy superfluous as
they had actually received their summonses via their substitutes as evidenced by their filing of
their pleadings, answers, and pre-trial brief. Such acts evidenced their voluntary appearance in
the action.

61. Office of the Court Administrator vs. Hon. Liberty O. Castaneda


A.M. No. RTJ-12-2316
Per Curiam

October 9, 2012

Facts:
Upon findings in a Two Judicial Audits and Physical Inventory of cases
conducted on the RTC of Paniqui, Tarlac, Branch 67, presided by respondent
judge, certain infirmities were found on how she handles cases on the said
sala. One of which is, In Several Cases, the Process server or sheriff merely
resorted to substituted service of summons without strict compliance with the
rule as well as established jurisprudence. Nonetheless, Judge Castaneda
acted on the petitions.
For Sheriff Collados defense, she admitted that she failed to observe the
requirements to validly effect substituted service as she was not aware
thereof and she was used to a pro-forma return of service but averred that it
was an honest mistake on her part and made assurances to strictly observe
the rules in the future service of summons.
Issue:
Whether or Not there was valid service of summons done by Sheriff Collado.
Ruling:
No, there was no valid of summons by Sheriff Collado.
For Substituted Service of Summons to be available, there must be several
attempts by the sheriff to personally serve the summons within a reasonable
period [of one month]which eventually resulted in failure to prove
impossibility of promt service. Several Attempts means at least three tries
preferably on two different dates. In addition, the sheriff must cite why such
efforts were unsuccesful. Only then can the impossibility of personal service
be confirmed or accepted.
62. Republic vs. Domingo
G.R. No. 175299, September 14, 2011
Leonardo-De Castro, J.
Facts:
On April 26, 2002, herein respondent Alberto A. Domingo filed a Complaint for
Specific Performance with Damages against the Department of Public Works
and Highways (DPWH), Region III, which was docketed as Civil Case No. 333M-2002 in the RTC of Malolos, Bulacan, Branch 18. Domingo averred that from
April to September 1992, he entered into seven contracts with the DPWH
Region III for the lease of his construction equipment to said government
agency.

Summons was served on DPWH Region III thru Nora Cortez, Clerk III.
Subsequently, on July 30, 2002, Domingo filed a Motion to Declare Defendant
in Default in view of the failure of the DPWH Region III to file a responsive
pleading within the reglementary period as required under the Rules of Court
which was granted by the court thereby resulting to a favorable decision in
favor of Domingo.
On August 27, 2003, the Republic of the Philippines, represented by the Office
of the Solicitor General (OSG), filed with the Court of Appeals a Petition for
Annulment of Judgment with Prayer for the Issuance of a Temporary
Restraining Order and/or a Writ of Preliminary Injunction, the Republic averred
that, under the law, the statutory representatives of the government for
purposes of litigation are either the Solicitor General or the Legal Service
Branch of the Executive Department concerned. Since no summons was
issued to either of said representatives, the trial court never acquired
jurisdiction over the Republic.
Issue:
Whether or Not, there was valid service of summons before DPWH Region III.
Ruling:
There was no valid service of summons before the DPWH Region III nor to the
Republic of the Phillipines.
In the instant case, the Complaint for Specific Performance with Damages
filed by Domingo specifically named as defendant the DPWH Region III. As
correctly argued by the Republic, the DPWH and its regional office are merely
the agents of the former (the Republic), which is the real party in interest in
Civil Case No. 333-M-2002. Thus, as mandated by Section 13, Rule 14 of the
Rules of Court, the summons in this case should have been served on the
OSG.
Thus, the court never acquired a valid jurisdiction over the DPWH or the
Republic of the Philippines.

You might also like